Saturday, January 07, 2017

Exchange with David Brightly

David Brightly: Science, science, science! Why is it such a bugbear? Does science have to be diminished in order to make room for faith? 

VR: Not if you make a distinction, as you and I both do, between science and scientism. The actual doing of science goes on with no problem without scientism, and the founding fathers of modern science, and some of the best practicioners today, are religious believers.

Suppose we take methodological naturalism to be a voluntary constraint on inquiry that rules out explanation and understanding in terms of persons. Science is then that body of understanding that eschews personhood as an explanatory factor. So there can be no science of world war one, say, and hence scientism is ruled out. Metaphysical naturalism becomes the doctrine that there are no persons other than the likes of us. Science then neither requires nor implies metaphysical naturalism, and there is plenty of space within naturalism for lines of inquiry that lie outside science.

VR: The only thing is that scientific enterprises get funded in ways that others do not.  But we have to ask what the scientific community is trying to accomplish. The scientific community can draw the limits of their own inquiry any way they choose. However, if they put something outside the realm of scientific inquiry, and then make heavy weather out of the fact that science hasn’t produced evidence for it, then we have a problem.  It’s no insult to a metal detector that it can’t find a $100 bill you might have left on the beach.

With this understanding of naturalism isn't it just a bit odd to speak of religious faith and 'faith in naturalism' in the same breath, as Lennox does? I would have thought that if someone's faith in Christ were on a par with my faith in naturalism it would amount to such a meagre, milksop kind of thing as to be not worth having. 

VR: But there are people out there with far more zeal and dedication to atheistic naturalism than a lot of Christians I know have with respect to their faith.  Atheism matters to these people, they want others to embrace it, and they are willing to deny access to positions of scientific or philosophical authority to those who disagree with their naturalism.

Surely the essence of much religion and certainly Christianity is the conviction that personhood lies at the very heart of things. Faith in Christ involves a relation with a person with all the emotional and moral implications that has. Atheists just don't feel this way.


I would agree in the sense that a Christian’s faith is a different kind of thing from faith in naturalism. On the other hand, I think it is epistemologically similar. On the other hand there are epistemological similarities. One considers the reasons for and against, and one commits to naturalism, or some religious view. Because a large part of a person’s life is structured around the decision one makes, it is understandable that people will be slow to reconsider their positions once taken. I do not see any less obstinacy of belief on either side of the issue. 

287 comments:

1 – 200 of 287   Newer›   Newest»
B. Prokop said...

"Surely the essence of much religion and certainly Christianity is the conviction that personhood lies at the very heart of things."

I would have expressed it a bit differently, but I totally agree with the essence of that statement. Amongst the Central Truths of Christianity is that the One God is a community of Persons (the Trinity). At its most fundamental core, reality is relationship. That's why Dante could write of "The Love that moves the Sun and the other stars." (my emphasis)

And it is also why all these internet debates and discussions about "the existence of God" or "proofs for God" are ultimately futile. As one rabbi (sorry, I can't remember his name) once said, "God is not a proposition to be proven. He is a Person to be encountered." (or words to that effect)

Although I do believe that it is possible to prove the existence of God, I maintain that doing so would be completely irrelevant without knowing Him. (Then you will begin to say, 'We ate and drank in your presence, and you taught in our streets.' But he will say, 'I tell you, I do not know where you come from.')

Victor Reppert said...

But, although intellectual assent to God is not saving faith, it is also pretty much impossible for lots of people if they think that what it is do be "bright" is to reject belief in God.

Unknown said...

I heard Hank Green, an apparently staunch atheist, say, "Science is a process, not an ideology." One tool of many. The problem may not be the recognition of the falsehood of scientism, but the recognition of its use. I know many atheists who simply never realize they're using this qualifier, and even if they do, simply naming it for what it is or giving them a definition of what they're advocating for will make them immediately realize the problem.

David Brightly said...

To say that the scientific community can draw the limits of their own inquiry is to concede too much. I'm sufficiently old-fashioned to think that the line of demarcation between science and non-science is clear enough. So it's not up to the scientific community to declare things in or out of bounds. Do you have a particular instance of this in mind?

Surely militant atheism, whether or not playing fast and loose with the scope of science, is part of a larger political movement seeking to impose itself. Eroding religious convictions is seen as a means of reducing political opposition. Likewise finagling academic appointments. Hardly setting a high value on free inquiry. Other than this, I can't see the point of being a proselytising atheist.

B. Prokop said...

I don't know who Hank Green is, but as for science being a process and not an ideology, he is dead on correct. It is a methodology, the use of which is sometimes appropriate, and sometimes wildly inappropriate.

I have a truly passionate interest in astronomy, and would never dream of approaching it from any standpoint other than that of the scientific method. However, I would also never dream of "going away" from it confined to the straitjacket of said methodology. For a good illustration of what I mean,
this is what I mean by "approaching" and this is what I mean by "going away".

Different tasks, different tools.

Unknown said...

"To say that the scientific community can draw the limits of their own inquiry is to concede too much. I'm sufficiently old-fashioned to think that the line of demarcation between science and non-science is clear enough. So it's not up to the scientific community to declare things in or out of bounds. Do you have a particular instance of this in mind?"

Right: I would define science by what it's been useful for, what it has worked for. It's a relative term, at this point, with everyone defining it by what they think makes the best tool for advancing towards truth. The atheistic pretensions of scientists today already has its effect, i.e. think of how much they won't study simply because it implies the unnatural, or of the numerous implausible cosmological models that immediately succeeded the proposal of the big bang--held on to far longer than they should have been simply because they avoided the conclusion of the "God-delusion". And since bias has become interchangeable with "delusion" by some of atheism's hateful popularizers, I suppose you could call them delusional for this.
(I say hateful, because, i.e.: https://www.google.com/url?sa=t&rct=j&q=&esrc=s&source=web&cd=1&cad=rja&uact=8&ved=0ahUKEwiip4myuLPRAhVGRSYKHfApAIgQtwIIHDAA&url=https%3A%2F%2Fwww.youtube.com%2Fwatch%3Fv%3DuPqqp8KVuQU&usg=AFQjCNGEmDOTsbalQX_QDXDfRexjgjYdvg&sig2=egXx_ILYG-YSgBpfxPHeFQ)


Unknown said...

A video of Bill Craig reasoning with a perfect example of scientism: https://www.youtube.com/watch?v=ZiVOhj6ehlw

Ilíon said...

B.Prokop: "And it is also why all these internet debates and discussions about "the existence of God" or "proofs for God" are ultimately futile."

VR: "But, although intellectual assent to God is not saving faith ..."

True, intellectual assent to the reality of God is not saving faith: the demons believe, and tremble. Intellectual assent to the reality of God is not sufficient to save one's life/existence, but it is an important step in getting there, and may perhaps be necessary.

VR: "... it is also pretty much impossible for lots of people if they think that what it is do be "bright" is to reject belief in God."

These interminable "arguments" are not wholly futile. Sure, the likes of Cat Metzger and Psychotic Dust will never admit the truth that atheism is the false view of reality, and likely neither will the likes of ... JJL. But, there are more people in the world (and reading) than committed ideologues-who-will-not-reason.

Joseph Hinman (Metacrock) said...

Part 2 of my assault on Draper and his why does God allow pain and suffering his indifference argument.

More on Draper

bmiller said...

@Ilíon,

^ I see what you did there ;-)

Ilíon said...

You saw one, surely. But did you see the other?

bmiller said...

@Ilíon,

I think so. I make typos. I can't recall you making any.

StardustyPsyche said...

B. Prokop said...

" I do believe that it is possible to prove the existence of God,"
Interesting. All attempts to prove the existence of god that I have ever read or heard fail immediately.

Would you be kind enough to lay out this proof I am presently ignorant of?


January 08, 2017 7:36 AM

StardustyPsyche said...

Jo F said...

" (I say hateful, because, i.e.: https://www.google.com/url?sa=t&rct=j&q=&esrc=s&source=web&cd=1&cad=rja&uact=8&ved=0ahUKEwiip4myuLPRAhVGRSYKHfApAIgQtwIIHDAA&url=https%3A%2F%2Fwww.youtube.com%2Fwatch%3Fv%3DuPqqp8KVuQU&usg=AFQjCNGEmDOTsbalQX_QDXDfRexjgjYdvg&sig2=egXx_ILYG-YSgBpfxPHeFQ)"
Hate the sin, love the sinner.


January 08, 2017 2:00 PM

StardustyPsyche said...

Jo F said...

" A video of Bill Craig reasoning with a perfect example of scientism: https://www.youtube.com/watch?v=ZiVOhj6ehlw"
Craig is correct about the rationality of accepting the unprovable postulates he lays out. What he fails to grasp is that it is equally rational to accept the unprovable conclusion that god does not exist.

Craig uses a double standard. By dismissing his opponents arguments as logically fallacious as proofs he calls for absolute proof of the things his opponent claims are rationally evidenced, yet he cannot produce an argument for god that is free of the very fallaciousness he uses to dismiss the arguments of his opponents.


January 08, 2017 2:02 PM

StardustyPsyche said...

Ilíon said...
"Psychotic Dust"
Good One!!! I have added it to my ever growing list of witty manglings of my handle:-)


January 08, 2017 9:40 PM

B. Prokop said...

"Would you be kind enough to lay out this proof I am presently ignorant of?"

I've done so many, many times, right here on this site. But perhaps I didn't qualify my statement enough. So here's the edited version: I do believe that it is possible to prove the existence of God to a person with an open mind, who has not prejudged the outcome of any argument as an "immediate fail".

For myself, the fact that there is something rather than nothing is all I need. Game, set, match. Next question?

StardustyPsyche said...

B. Prokop said...

"Would you be kind enough to lay out this proof I am presently ignorant of?"

" I've done so many, many times, right here on this site."
Pity I did not see any such argument on this site from you or anybody else.

" But perhaps I didn't qualify my statement enough. So here's the edited version: I do believe that it is possible to prove the existence of God to a person with an open mind, who has not prejudged the outcome of any argument as an "immediate fail"."
Oh good, then you can present it to me without worry, since I began the request by allowing for my own apparent ignorance.

" For myself, the fact that there is something rather than nothing is all I need. "
Rrrrr...clunk, cough, sputter, die.
Darn, another one bites the dust.

Ok, I will help you out, that is a non-sequitur, suffering from the false dichotomy between god and absolutely nothing at all.

If god exists god has to be some thing else it is no thing. Since god is some thing then it is possible for a thing to exist eternally or outside of time or to pop into existence ex nihilo. Since your particular version of god is a particular unevidenced speculated thing then it is equally likely that any of an unbounded number of alternative specifically formulated speculated things have by themselves the capability to give rise to our observable universe or that they do so only in particular speculated combinations of interactions with other speculated things. Thus, your formulation of god, rather than being necessary, is just an infinitesimally likely speculation among an unbounded number of such speculations.

Further, your formulation of god has no explanatory power, only asserting irrationalities in an attempt to solve irrationalities.

Perhaps the simplest alternative to your god is the one we have vast evidence for via the observation of conservation, that the stuff matter/energy is made of is eternal and our ability to cope rationally with this eternal existence is our problem, not the problem of eternal stuff.

"Game, set, match. Next question?"
Yes, when are you going to provide a proof of the existence of god that does not fail immediately?


January 13, 2017 12:53 PM

B. Prokop said...

I didn't expect it to convince you. The fact that it hasn't is utterly irrelevant to the strength of the argument, or to its explanatory power. Your response, laden with misinterpretations, bizarre suppositions, and logical fallacies, only demonstrates to any objective observer that you are not a serious participant in these discussions.

(Now that's a statement that's ludicrously easy to prove, simply by reading your various postings.)

"Yes, when are you going to provide a proof of the existence of god that does not fail immediately?"

I just did.

StardustyPsyche said...

B. Prokop said...

" I didn't expect it to convince you. The fact that it hasn't is utterly irrelevant to the strength of the argument, "
Indeed, the argument fails due to its lack of merit as I outlined above. I made no argument from personal incredulity so your response is a straw man.

" Your response, laden with misinterpretations,"
Such as?

"bizarre suppositions, "
Such as?

"and logical fallacies,"
Such as?

" only demonstrates to any objective observer that you are not a serious participant in these discussions."
Since you did not enumerate any specific defects I can only characterize your assertion of them as vacuous.


"Yes, when are you going to provide a proof of the existence of god that does not fail immediately?"
" I just did."
Your "argument" is that existence proves god, which of course is preposterous for at least the reasons I provided above.


January 13, 2017 1:46 PM

B. Prokop said...

OK, but be forewarned. I have no intention of indulging your predilection for running down rabbit holes. I'll demonstrate that there is nothing vacuous about my assertions, and unless you show at least some sign of understanding the issue here, I'll leave the field to you. I'll not blow dust up in my eyes forever trying to fan a flame where there is no spark to work from.

Misinterpretations:

- "false dichotomy between [G]od and absolutely nothing at all"

Nope. Nothing false about it. Those are your choices - there is no third.

- "alternative specifically formulated speculated things have by themselves the capability"

Sorry, but God is neither "specifically formulated" or even a "thing". Your reasoning may be relevant to something else, but certainly has no bearing on any rational discussion of God.

Bizarre suppositions:

- "[G]od, rather than being necessary, is just an infinitesimally likely speculation among an unbounded number of such speculations."

Just proves that, after 400 plus comments below, you still haven't got a clue as to the meaning of "necessary" and "contingent".

Just where do you come to your conclusion (or any conclusion) about the "likelihood" of there being a Creator, that you can assign any probability to it whatsoever, let alone "infinitesimal"? That supposition (that one can assign probabilities in this matter) is truly bizarre.

Logical fallacies:

- "no explanatory power"

Your rejection of the explanation(s) has no bearing on the fact that acknowledgement of the Creator God explains, not only much, but pretty much everything.

- "observation of conservation"

Not even under discussion, yet you (once again) drag it in.

Now. The ball is in your court. You can either show some sign of genuine interest in the issue at hand, or you can revert to your default position of dismissing everything out of hand. Your choice. Just don't expect a response if your choice is obfuscation and invincible ignorance. In that event, the floor is yours, and you're welcome to it.

StardustyPsyche said...

Blogger B. Prokop said...

"least some sign of understanding the issue here, I'll leave the field to you."
I understand that the very language of theological analysis is fallacious and built upon muddled notions of cause and effect as well as a great many other conceptual defects, but by all means, give it a go.


" - "false dichotomy between [G]od and absolutely nothing at all"

Nope. Nothing false about it. Those are your choices - there is no third."
Except for all the alternatives I enumerated above January 13, 2017 1:36 PM, so your assertion is utterly fallacious and a demonstrated false dichotomy.


" - "alternative specifically formulated speculated things have by themselves the capability"

" Sorry, but God is neither "specifically formulated" or even a "thing". "
You assert a particular sort of god among a large number of asserted sorts of god and among an unbounded number of potential sorts of deistic or theistic assertions.

Further, god must be some thing, else god is no thing. God is something or god is nothing. Since nothing has no properties or powers or anything else that leaves the only choice that god is something, some thing, which is clearly a thing.

Basic language skills.

"Your reasoning may be relevant to something else, but certainly has no bearing on any rational discussion of God."
I have not noticed you applying any great depth of rationality here. I mean, you think god is something but not a thing. How rational is that?

" Bizarre suppositions:

- "[G]od, rather than being necessary, is just an infinitesimally likely speculation among an unbounded number of such speculations.""
That was a conclusion based on argument, not a supposition. How do you claim to be arguing rationally when you fail to differentiate between a supposition and a conclusion?

" Just proves that, after 400 plus comments below, "
Uhm, I see the counter at 21, I suppose this will be 22. How rational is it to conflate these things? I guess you are referring to the other thread.


January 13, 2017 5:49 PM

StardustyPsyche said...


Blogger B. Prokop said...


"you still haven't got a clue as to the meaning of "necessary" and "contingent"."
Necessary means something that must be and is therefore original. Contingent is something that depends upon or is a result of something else.

There are no sound arguments for a theistic god as a necessary being that have ever been published into general circulation. Your assertion of mere observation of existence as one such argument is preposterous.

" Just where do you come to your conclusion (or any conclusion) about the "likelihood" of there being a Creator, that you can assign any probability to it whatsoever, let alone "infinitesimal"?"
I explained that above. Your formulation of the god speculation is presumably Christian, which is incoherent owing to the assertion of mutually exclusive properties. A deistic formulation of the speculation of god does not suffer from being incoherent, merely from being idle speculation.

"Infinitesimal" is a somewhat murky term but is sometimes thought of as being unboundedly small yet non-zero. If a set has an unbounded number of members each member is an infinitesimal part of that set. There are an unbounded number of potential idle speculations of which any particular speculation of god is just one.

What is the probability that the universe was created 5 seconds ago with the appearance of age? You cannot prove the universe was not created 5 seconds ago with the appearance of age, but given the purely speculative nature of that assertion it is rational to consider such a creation as infinitesimally likely.

A non-disprovable idle speculation, god for example, is rationally considered to be infinitesimally likely.

" Logical fallacies:
- "no explanatory power"
Your rejection of the explanation(s) has no bearing on the fact that acknowledgement of the Creator God explains, not only much, but pretty much everything."
You have it back to front. I suggest you study what the term means. This has nothing to do with my personal rejection. The speculation of god fails due to lack of merit and therefore fails to meet the conditions of explanatory power. You can read what those conditions are here:
https://en.wikipedia.org/wiki/Explanatory_power


" - "observation of conservation"
Not even under discussion, yet you (once again) drag it in."
It is an alternative to your false dichotomy. How rational are you to not understand this position in my argument?

" Now. The ball is in your court. You can either show some sign of genuine interest in the issue at hand, or you can revert to your default position of dismissing everything out of hand."
Your arguments fail immediately, but not because I say so, rather, for the many reasons I have given.

Are you accusing me of avoidance brevity? Really?


January 13, 2017 5:49 PM

B. Prokop said...

My point having been made, I leave the floor to you. Have at it.

bmiller said...

@B. Prokop,

It shouldn't be surprising that he doesn't think God really exists. After our last discussion I doubt if you could convince him you really exist (unless you're a quark maybe).

SP,

SP:"I understand that the very language of theological analysis is fallacious and built upon muddled notions of cause and effect as well as a great many other conceptual defects, but by all means, give it a go."

You've made some assertions here: your opponent uses "fallacious language", his notions of cause and effect are muddled, there are many other "conceptual defects".

Although I'm really interested in what you mean by "fallacious language". Why don't you un-muddle us poor souls on our notions of cause and effect?




Kevin said...

"There are no sound arguments for a theistic god as a necessary being that have ever been published into general circulation."

Of course there are. Your failure to grasp them or recognize them is irrelevant to the fact that such arguments exist. They were presented to you in the other thread, over and over, and you remained utterly incapable of, or unwilling to, understand them. Your attempts at refutation failed immediately, but not because I say so. Just a simple truth, independent of my opinion.

Joseph Hinman (Metacrock) said...

Stardust:

"There are no sound arguments for a theistic god as a necessary being that have ever been published into general circulation."


here, this is one. It's by little unknown Philosopher named Charles Hartshorne

g --> N(g)
N(g) v ~N(g)
~N(g) --> N(~N(g))
N(g) v N(~N(g))
N(~N(g)) --> N(~g)
N(g) v N(~g)
~N(~g)
N(g)
N(g) --> g
g


answer it, show me the fallacy.

Ilíon said...

one of the (obviously trolling) fools: "[G]od, rather than being necessary, is just an infinitesimally likely speculation among an unbounded number of such speculations"

B.Prokop: "Just proves that, after 400 plus comments below, you still haven't got a clue as to the meaning of "necessary" and "contingent"."

one of the (obviously trolling) fools: "no explanatory power"

B.Prokop: "Your rejection of the explanation(s) has no bearing on the fact that acknowledgement of the Creator God explains, not only much, but pretty much everything."

one of the (obviously trolling) fools: "There are no sound arguments for a theistic god as a necessary being that have ever been published into general circulation."

LoL: "Of course there are. Your failure to grasp them or recognize them is irrelevant to the fact that such arguments exist. They were presented to you in the other thread, over and over, and you remained utterly incapable of, or unwilling to, understand them."

And so on, ad nauseam et ad infinitum.

When will all of you people reading this thread, and especially you "nice" people, get it into your minds that these fools are not different in kind from every other so-called atheist you ever encountered or ever will encounter? These fellows are "trolls", sure; but that is a different matter from their God-hatred (however that the trolling may be linked to and fed by the God-hatred).

The difference -- the only difference that matters -- between the likes of Cat Metzger and Psychotic Dust on the one hand, and JJL on the other, is that they openly trumpet their refusal to reason soundly and to acknowledge that not only are there sound-and-undefeated arguments for the reality of the Creator but that there are *no* sound, much less undefeated, arguments for God-denial.

All the evidence, of every sort, is on the side of the reality of God and against God-denial. And how does *every* God-denier, including JJL, respond to that fact? With something that amounts to: "[G]od, rather than being necessary, is just an infinitesimally likely speculation among an unbounded number of such speculations" or "no explanatory power" or "There are no sound arguments for a theistic god as a necessary being that have ever been published into general circulation."

Goet it into your minds: every God-denier is intellectually dishonest, else he would acknowledge that God is (and that he is not God)

Ilíon said...

[to continue]

Atheists -- all of them, including JJL -- are playing a game which I call "Deny and Demand".

It doesn't matter which argument for the reality of the Creator-God that you present, nor how well you present it, nor how thoroughly you trounce their feeble attempts at countering it, in the end they will simply deny that you have made the case (*) and demand a "better" argument.

And, instead of responding with something like "Your rejection of the explanation(s) has no bearing on the fact that acknowledgement of the Creator God explains, not only much, but pretty much everything", most of you willingly fall for "Deny and Demand", every time.

(*) the reasoning, such as it is, is something like this: You have not *forced* me to acknowledge that my God-denial is the false view of reality, ergo there is no God But, of course, their freedom is one of the major points of evidence for the reality of the Creator.

StardustyPsyche said...

B. Prokop said...

" My point having been made, I leave the floor to you. Have at it."
Yes, you have made the point that you have no demonstrated capacity for rational engagement.

You have some muddled thoughts about cause and effect that are unable to withstand immediate refutation so you just ad hoc declare that the other person doesn't understand anything and you walk away.

Does that somehow make you feel satisfied that you have some kind of rational capacity for logical argumentation?


January 13, 2017 7:23 PM

Unknown said...

Legion: "Of course there are [sound arguments for a theistic god as a necessary being that have ever been published into general circulation]. Your failure to grasp them or recognize them is irrelevant to the fact that such arguments exist. They were presented to you in the other thread, over and over, and you remained utterly incapable of, or unwilling to, understand them."

Okay. Please copy and paste, from the other thread to, the "sound arguments for a theistic god as a necessary being that have ever been published into general circulation."

You have asserted that this was provided over and over on the other thread, so let's see if what you write is true or false.

Just copy and paste, from the other thread, the very best example from the "sound arguments for a theistic god as a necessary being that have ever been published into general circulation."

Or is it possible that you and the other apologists have only false assertions to repeat to yourselves now? Is that what you guys are left with, entirely? Because that sure is what this thread reads like.

Legion: "Your attempts at refutation failed immediately, but not because I say so. Just a simple truth, independent of my opinion."

Failed immediately how? Demonstrate.

Stardusty has quickly and efficiently demonstrated (as in using logic, and actually avoiding obvious fallacies) how these various arguments fail.

Why do you suppose it will be so hard for you do what I ask above?

Is it possible that the problem lies not with you, but with the belief you're trying to defend?

Joseph Hinman (Metacrock) said...

When will all of you people reading this thread, and especially you "nice" people, get it into your minds that these fools are not different in kind from every other so-called atheist you ever encountered or ever will encounter? These fellows are "trolls", sure; but that is a different matter from their God-hatred (however that the trolling may be linked to and fed by the God-hatred).

we know. atheis suck,we hate atheists, yeah us,ra ra! thanks for yiour contribution genius!

Joseph Hinman (Metacrock) said...

Yes, you have made the point that you have no demonstrated capacity for rational engagement.

You have some muddled thoughts about cause and effect that are unable to withstand immediate refutation so you just ad hoc declare that the other person doesn't understand anything and you walk away.

Does that somehow make you feel satisfied that you have some kind of rational capacity for logical argumentation?


Star Dust I don't see you answer in the argument, since you know it all show us what's wromng with the modal logic,I am waiting

Joseph Hinman (Metacrock) said...

Cal the magnificent (skeptic?) says:




You have asserted that this was provided over and over on the other thread, so let's see if what you write is true or false.

I just just put one up there, all g stuff. That's called modal logic it's an actual argument. Still waiting to see it answered, Charles Hartshorne is considered one of the major thinkers of the 20th century,Just because you don't know about him doesn't mean he's not, so answer it!

Just copy and paste, from the other thread, the very best example from the "sound arguments for a theistic god as a necessary being that have ever been published into general circulation."

here:

g --> N(g)
N(g) v ~N(g)
~N(g) --> N(~N(g))
N(g) v N(~N(g))
N(~N(g)) --> N(~g)
N(g) v N(~g)
~N(~g)
N(g)
N(g) --> g
g


Or is it possible that you and the other apologists have only false assertions to repeat to yourselves now? Is that what you guys are left with, entirely? Because that sure is what this thread reads like.

STOP IGNORING ME IDIOT AMD ANSWER THE BLEEDING ARGUMENT YOU SHOW OFF KNOW NOTHING

Joseph Hinman (Metacrock) said...

Have Theology--Will Argue

wire Metacrock

Joseph Hinman (Metacrock) said...

Goet it into your minds: every God-denier is intellectually dishonest, else he would acknowledge that God is (and that he is not God)

Probably, I've been doing this apologetic stuff for over 15 years and posted with thousands of atheists, I have done over half a million posts,I think I know what they are like,

Right now in my life and in the history of this country I can't say that much for the god accepters either,thanks for helping. can you stop now?

StardustyPsyche said...

bmiller said...

" @B. Prokop,

It shouldn't be surprising that he doesn't think God really exists. After our last discussion I doubt if you could convince him you really exist"
Indeed, among the unbounded alternative speculations to Prokop's false dichotomy is that I am god and you are all figments of my divine imagination. Descartes and many others realized this is a non-disprovable speculation. This rather egocentric speculation is as valuable and rational as any other speculation of god.

" (unless you're a quark maybe)."
We are all made of quarks, as well as other things presently considered as fundamental.

" You've made some assertions here: your opponent uses "fallacious language", his notions of cause and effect are muddled, there are many other "conceptual defects".

Although I'm really interested in what you mean by "fallacious language". Why don't you un-muddle us poor souls on our notions of cause and effect?"
I already have, many times, but to briefly recap the assignment of the title of "cause" to an object is hopelessly muddled.

Returning to WL Craig, and these are his words, not yours, so not necessarily your burden, but they are so typical of how muddled theistic language is even for a man of great education, a man of no few letters, yet so terrible foolish in his concepts and language on this subject.

He gave a lecture attempting to demonstrate the rationality of a cause and effect being outside of time in order to justify the incoherent assertion that a god outside of time acts in a temporal sequence of events.

In this lecture he used the example of a ball on a pillow. He said the ball is the cause of the depression in the pillow. The ball and the depression exist simultaneously and therefore cause and effect can exist simultaneously and therefore cause and effect can exist in zero time therefore timelessly and therefore a timeless god who is a cause is not an incoherent concept.

Rubbish.

The ball is not a static object, rather, a seething stew of frenetic activity. The molecules, atoms, protons, neutrons, electrons, up quarks and down quarks are moving about at high speed, all bouncing about in a continuous or continual system of temporal interactions in which cause and effect are arbitrary. When 2 atoms collide both are the cause and both are the effect of each other. To assign the title of "cause" to one and "effect" to the other is arbitrary.

And the same with the pillow. And the same with the interface between the ball and the pillow. And the same with the entire planet Earth, our solar system, and our big bang we live in, all of which are linked by fields such as gravity continuously or continually moving in a vastly complex temporal system.

The ball and the pillow are not static objects, they are dynamic systems always in change in a never ending temporal system.

The assignment of the title of "cause" to the ball and "effect" to the pillow is hopelessly muddled and simplistic thinking and in no way demonstrates that a cause and effect can exist simultaneously and therefore cause and effect can proceed atemporally. The ball/pillow system is an inherently temporal system that Craig has failed to understand by using his muddled language.

Further, Craig fails to recognize that he has only provided a rationale for eternal matter/energy. If cause and effect can exist simultaneously and atemproally for ordinary objects such as a ball and a pillow then such objects can be eternal and there is no problem of first cause.

But Craig is wrong by virtue of his primitive concepts of assigning the titles of "cause" and "effect" to our macro approximation concepts of objects, so the riddle of an actual infinite for the origins of existence remains unsolved.


January 13, 2017 8:39 PM

StardustyPsyche said...

Legion of Logic said...

" "There are no sound arguments for a theistic god as a necessary being that have ever been published into general circulation."

Of course there are. Your failure to grasp them or recognize them is irrelevant to the fact that such arguments exist. They were presented to you in the other thread, over and over,"
You mean Aquinas? Please don't make me laugh. I pointed out the fallacies easily.

Aquinas has been shown to be fallacious again and again.


January 13, 2017 9:17 PM

B. Prokop said...
This comment has been removed by the author.
B. Prokop said...

Joe,

You are quite wrong in criticizing Ilion's post. He is basically reminding us of the very advice of Christ Himself (and you can never go wrong in following the Master):

"Do not throw your pearls before swine, lest they trample them under foot and turn to attack you." (Matthew 7:6)

"And if any one will not receive you or listen to your words, shake off the dust from your feet as you leave that house or town." (Matthew 10:14)

Or this, from St. Paul:

"If any one does not agree with the sound words of our Lord Jesus Christ and the teaching which accords with godliness, he is puffed up with conceit, he knows nothing; he has a morbid craving for controversy and for disputes about words."
(1 Timothy 6:3) Sound familiar?

Or this, from St. Augustine's City of God, Book II (deserves careful reading):

"Even after the plain truth has been thoroughly demonstrated, so far as a person is capable of doing, the confirmed skeptic will insist on maintaining belief in his own irrational notions. This is due to either a great blindness, which renders him incapable of seeing what is plainly set before him, or on account of an opinionative obstinacy, which prevents him from acknowledging the truth of what he does see. Thence arises the woeful necessity of going to ridiculous lengths to expound yet more fully on what we have already made perfectly clear, in hopes that we might get through to those who close their minds to reason.

And yet how shall we ever profit from our discussions, or what bounds can be set to our discourse, if we forever fall to the temptation of replying to those who reply to us? We must acknowledge that those who are so hardened by the habit of contradiction will never yield, but would rather reply out of stubbornness, even when they recognize their own error."

And I'll close with this gem from C.S. Lewis (this is a blog about Lewis, after all):

"If there is a real [intellect] - even the least trace of one still there inside [all the nonsense], it can be brought to life again. If there's one wee spark under all those ashes, we'll blow till the whole pile is red and clear. But if there's nothing but ashes we'll not go blowing them in our own eyes forever."

StardustyPsyche said...

Joe Hinman said...

Stardust:
"There are no sound arguments for a theistic god as a necessary being that have ever been published into general circulation."

" here, this is one. It's by little unknown Philosopher named Charles Hartshorne

g --> N(g)
N(g) v ~N(g)
~N(g) --> N(~N(g))
N(g) v N(~N(g))
N(~N(g)) --> N(~g)
N(g) v N(~g)
~N(~g)
N(g)
N(g) --> g
g

answer it, show me the fallacy."
First, you would have to define your terms. What is g? How does N qualify g?

Hartshorne tried to revive the ontological argument, which has been debunked for centuries even by Aquinas, Kant, and many others.

Pretty lame, but if you really want to go through it by all means, give it a go, but the problem is that just because you can imagine it does no make it so. How anybody can take such a thing seriously for more than a minute I have no idea.


January 13, 2017 11:47 PM

StardustyPsyche said...

Ilíon said...

" Goet it into your minds: every God-denier is intellectually dishonest, else he would acknowledge that God is (and that he is not God)"
The speculation that I am god has equal rational validity to any other particular formulation of the speculation of god.

For me to say I am not god I can only argue from personal incredulity, I cannot actually disprove the assertion that I am god. The best I can do is say that I do not feel like I am god, and it seems ridiculous that I am god.

But I cannot prove and you cannot prove that you are not mere figments of my divine imagination.

I merely choose, ad hoc, to provisionally accept the basic reliability of the human senses and live my life on that provisional basis.


January 14, 2017 4:26 AM

StardustyPsyche said...

B. Prokop said...

You are quite wrong in criticizing Ilion's post. He is basically reminding us of the very advice of Christ Himself (and you can never go wrong in following the Master):

"Do not throw your pearls before swine, lest they trample them under foot and turn to attack you." (Matthew 7:6)"
Be fishers of men.
Love your neighbor as yourself.
Love your enemy as yourself.
Judge not lest yee be judged.
Let he among you without sin cast the first stone.
(I will trust our resident bible experts to provide the chapters and verses)

January 14, 2017 9:52 AM

B. Prokop said...

... and right on cue, proving my point.

bmiller said...

@Stardusty Psyche,

SP:"Although I'm really interested in what you mean by "fallacious language". Why don't you un-muddle us poor souls on our notions of cause and effect?"
I already have, many times, but to briefly recap the assignment of the title of "cause" to an object is hopelessly muddled.

Returning to WL Craig, and these are his words, not yours, so not necessarily your burden, but they are so typical of how muddled theistic language is even for a man of great education, a man of no few letters, yet so terrible foolish in his concepts and language on this subject."

As you note, I am not WL Craig, I'm not making that argument and I don't care about that argument.
I'm interested in challenging your apparent view that cause and effect are illusions, but the details of your position are not clear to me. I can't start to criticize it if I don't understand it. I welcome your detailed explanation of that position, but if that is not forthcoming, I have some questions.

So my questions are (in no particular order):
1) Do you believe that cause and effect are an illusion?
2) Do you believe that you or others can cause an effect?
3) How would you define "cause" and "effect" if not illusions?
4) Do you believe that things actually exist at all? By that, I mean does Bob Prokop exist as an actual entity rather than just an arbitrary boundary of quarks or whatever you postulate as the fundamental thing?
5) How do we come to know how the universe changes, if indeed you hold that it does in fact change?

B. Prokop said...

"Does Bob Prokop exist as an actual entity rather than just an arbitrary boundary of quarks or whatever?"

Hey, there's nothing arbitrary about my boundaries!

Joseph Hinman (Metacrock) said...

. Prokop said...
Joe,

You are quite wrong in criticizing Ilion's post. He is basically reminding us of the very advice of Christ Himself (and you can never go wrong in following the Master):


I commend your concern Prokop but I thought about that idea a lot in those 15 years, I think these are not my pearls. I agree that no point in trying to discuss serious ideas with idiots. The atheists we deal with here are not idiots if you think they are the worst atheist you really are shaltered, there are much worse atheists than these.

Indion as much as said don't try to talk to any atheist, he chased Lowder off the board by being a jerk and Lowder is not at all in That category,Lowder is willing to talk and he considers the arguments I make

bmiller said...

Hey, there's nothing arbitrary about my boundaries!

Ha Ha! My boundaries have actually grown over the last month.

B. Prokop said...

"there are much worse atheists than these"

Oh, I quite agree. If you can stomach it, spend some time on any random comments section over on The Guardian, if the article in any way deals with religion. Pure, unadulterated hatred and bile, expressed in stupefyingly repetitive postings. There are no atheists like British atheists (and I oughta know - I lived there for several years). They make ISIS look positively moderate in their expressed desire to annihilate anyone who disagrees with them.

Joseph Hinman (Metacrock) said...

answer it, show me the fallacy."
First, you would have to define your terms. What is g? How does N qualify g?

I think those are standard

Hartshorne tried to revive the ontological argument, which has been debunked for centuries even by Aquinas, Kant, and many others.

He did not try to revive it. this proves you are totally non conversant with the history of the argument,I suggest you read the book The Many faced Argument by John Hick.

were you read on the subject you would realize that mo modern analytical philosopher think's Heartshorne's argument is victim to the same answers that Kant too to it. his use of s5 modal Logic was a total game changer,stop turning to atheist websites for answers and start reading real philosophers.BTW saying that is not an answer, you haven not given me a fallacy.




Pretty lame, but if you really want to go through it by all means, give it a go, but the problem is that just because you can imagine it does no make it so. How anybody can take such a thing seriously for more than a minute I have no idea.

you are truly ignorant, you not even wrong, you so far out of the loop. But hey you still have not made an argument. Calling arguments names is not an argument, you have said nothing,we can insult the beast more. Let's say it's pathetic and lame. .let's go
Shakespeare on it and say it's n Elfin skinned mumble bailey and a flap dragon that does not show us a fallacy

Joseph Hinman (Metacrock) said...

Stardust

"N(A)" means "it is logically necessary that A," "~A" means "it is not the case that A," "-->" is strict implication, "v" means "or," and "g" means "God exists":

Joseph Hinman (Metacrock) said...

Oh, I quite agree. If you can stomach it, spend some time on any random comments section over on The Guardian, if the article in any way deals with religion. Pure, unadulterated hatred and bile, expressed in stupefyingly repetitive postings. There are no atheists like British atheists (and I oughta know - I lived there for several years).

I know been there done that.that comes under the heading of this is what I meant when I said "there are worse atheists than the one's here."

Joseph Hinman (Metacrock) said...

Stardust it just occurred to me you may not know what valid means, you think valid means true right? not exactly not in formal logic.

StardustyPsyche said...

bmiller said...

" I have some questions."
1) Do you believe that cause and effect are an illusion?
The assignment to an apparently static object the title of "cause" is an illusion. Cause and effect are mutual in a temporal process system.

2) Do you believe that you or others can cause an effect?
With absolute certainty? No. The the true details of a dynamic system are too complex for humans to quantify precisely and even in principle we are limited by the wavelength of our "measurement" "tool".

3) How would you define "cause" and "effect" if not illusions?
As logical concepts in idealized thought experiments and approximation models.

4) Do you believe that things actually exist at all? By that, I mean does Bob Prokop exist as an actual entity rather than just an arbitrary boundary of quarks or whatever you postulate as the fundamental thing?
There must be an existence of some sort as opposed to absolutely nothing at all. On the provisional acceptance of the basic reliability of the human senses we can function by defining objects as contained within their apparent borders, which is rational means to model reality and do useful work.

5) How do we come to know how the universe changes, if indeed you hold that it does in fact change?
To function we must first accept a number of postulates, either provisionally or as articles of faith. We who are scientifically minded accept them provisionally.

Among these provisionally accepted postulates is the basic reliability of the human senses which we use to observe change.


January 14, 2017 11:01 AM

StardustyPsyche said...


Blogger Joe Hinman said...

SP-First, you would have to define your terms. What is g? How does N qualify g?

" I think those are standard"
Non-responsive.


" were you read on the subject you would realize that mo modern analytical philosopher think's Heartshorne's argument is victim to the same answers that Kant too to it. his use of s5 modal Logic was a total game changer,"
Ha Ha Ha.

" you haven not given me a fallacy."
You have not defined your terms or the quantifying function.


January 14, 2017 11:56 AM

bmiller said...

@Stardusty Psyche,

Thanks for the answers.
I think wrt to Bob Prokop, you are willing to accept him as an existent entity, not with any type of certainty, but as "close enough for government work" (wink to Bob's pre-retirement life :-))

Now regarding cause and effect:

Me:"1) Do you believe that cause and effect are an illusion?
SP:"The assignment to an apparently static object the title of "cause" is an illusion. Cause and effect are mutual in a temporal process system."
You distinguish between "apparently static objects" and those in a "temporal process system". But in both cases you seem to imply that "cause" is an strange term. Since in the first case you consider it an illusion and in the other, it is meaningless in the ordinary sense of the word. It seems you are saying that effects cause causes in the same way that causes cause effects. Perhaps you mean that we shouldn't use those words at all.


Me:"2) Do you believe that you or others can cause an effect?"
SP:"With absolute certainty? No. The the true details of a dynamic system are too complex for humans to quantify precisely and even in principle we are limited by the wavelength of our "measurement" "tool"."
You see. Here you seem to understand cause in the ordinary sense of the term. The answer to question 1) implies that cause is a meaningless term altogether. How could one even in principle activate a cause that results in an effect from your answer to question 1)?

Me:"3) How would you define "cause" and "effect" if not illusions?"
SP:"As logical concepts in idealized thought experiments and approximation models."
How are "logical concepts" and "idealized thought experiments" different from "illusions"?

StardustyPsyche said...


Joe,
You still have not defined what god is, so you still have not defined your terms.

Nevertheless, just to play along with the silly idea that thinking makes it so, I did a word substitution based on what little you have provided so far.


IF God exists THEN it is logically necessary that(God exists)
it is logically necessary that(God exists) OR it is not the case that it is logically necessary that(God exists)
IF it is not the case that it is logically necessary that(God exists) THEN it is logically necessary that(it is not the case that it is logically necessary that(God exists))
it is logically necessary that(God exists) OR it is logically necessary that(it is not the case that it is logically necessary that(God exists))
IF it is logically necessary that(it is not the case that it is logically necessary that(God exists)) THEN it is logically necessary that(it is not the case that God exists)
it is logically necessary that(God exists) OR it is logically necessary that(it is not the case that God exists)
it is not the case that it is logically necessary that(it is not the case that God exists)
it is logically necessary that(God exists)
IF it is logically necessary that(God exists) THEN God exists
God exists


I will just start at the beginning
"IF God exists THEN it is logically necessary that(God exists)"
Only in the tautological sense that this statement applies to all existent things. If a thing exists then it is necessary that it exists since it is existent. That makes god nothing special.

If you mean some notion of alternative possibilities that makes god special necessarily then no, one can speculate that something gave rise to god, god's god, but maybe god's god died, though previously greater than god, but now dead, so now god exists.

The speculative alternative formulations are unbounded, hence the assertion of necessity is false.

Fail from the git go, but then, you did not fully define your terms so you might think you have some definitional alternatives to these failings.


Ilíon said...

Or this, also from St. Paul: "For since the creation of the world God's invisible qualities--his eternal power and divine nature--have been clearly seen, being understood from what has been made, so that people are without excuse."

According to the inspired Scriptures, any one who denies the reality of the Creator, does so willingly and obstinately; that is, it is not failure to believe in the Creator, but rather refusal to admit. According to the inspired Scriptures, no one has any excuse for refusing to admit the reality of the Creator.

The "strong" version (*) of the AfR is, among other things, a logical/rational demonstration of the truth of Paul's assertion in Romans 1:20.

(*) that is, my version rather than VR's

StardustyPsyche said...

bmiller said...


" Me:"3) How would you define "cause" and "effect" if not illusions?"
SP:"As logical concepts in idealized thought experiments and approximation models."
How are "logical concepts" and "idealized thought experiments" different from "illusions"?"
Indeed. Is, for example, a line and illusion? Perhaps.

Have you ever done any work with finite element analysis by any chance? A matrix of differential expressions is defined as starting at some state. With each iteration the state of each element changes according to the transfer function from the adjacent elements.

Thus, everything is a cause and everything is an effect. On determinism the future is determined by the present state and the transfer functions of each element. Everything is the cause of the next state and everything is an effect of the previous state.

But cause and effect are transitory, not a retained title. Cause and effect occur as a vast matrix interacting moment to moment in a temporal process.

So, the question of first cause, or first mover is still valid. How did this vast matrix of temporal interactions get started? Did it always exist? That would require an actual infinity, which is irrational. Did it just start moving ex nihilo? That is also irrational, just poof, that is magical thinking. Was there an intelligent being that got it all going? That doesn't help because we must ask all the same questions about this magical being, mental gymnastics to define one's way around these questions notwithstanding.

Hence the great existential riddle unsolved.

January 14, 2017 1:12 PM

bmiller said...

@Stardusty Psyche,
Will I get an answer to our exchanges on 1) and 2)?
I really want to understand your point of view on these 2.

" Me:"3) How would you define "cause" and "effect" if not illusions?"
SP:"As logical concepts in idealized thought experiments and approximation models."
Me:”How are "logical concepts" and "idealized thought experiments" different from "illusions"?"
SP:”Indeed. Is, for example, a line and illusion? Perhaps. “

This is what I want you to clear up with me regarding your position. Do you think how we can know how reality works? Is the universe intelligible?

Of course mathematics is very useful in our investigation of the physical world. But the abstract science of mathematics is not the same as the science of the physical world. Differential equations allow infinite series to exist, which I know, sends you up the wall since you fume at “actual infinities” of any sort. It seems odd that you would appeal now to the concept of infinite series as the basis of reality.

It makes no sense to me for me to engage you any further until I understand what exactly we agree on and what we have to dispute.

Joseph Hinman (Metacrock) said...

Stardusty Psyche said...

Blogger Joe Hinman said...

SP-First, you would have to define your terms. What is g? How does N qualify g?

" I think those are standard"
Non-responsive.

I answered in the next post. g = God exists.


" were you read on the subject you would realize that mo modern analytical philosopher think's Heartshorne's argument is victim to the same answers that Kant too to it. his use of s5 modal Logic was a total game changer,"
Ha Ha Ha.

" you haven not given me a fallacy."

yes I did I defined all the temrs i the modal argument that I posted,


"N(A)" means "it is logically necessary that A," "~A" means "it is not the case that A," "-->" is strict implication, "v" means "or," and "g" means "God exists":

bmiller said...

Do you think how we can know how reality works?
S/B
Do you think we can know how reality works?

Joseph Hinman (Metacrock) said...

Stardusty Psyche said...

Joe,


Now you are asking a theological question. since I just posted a modal argument using the signifies "g" I thought that;what you were asking for when asked the signification of "g." You should be more specific.

[edit]




I will just start at the beginning
"IF God exists THEN it is logically necessary that(God exists)"
Only in the tautological sense that this statement applies to all existent things. If a thing exists then it is necessary that it exists since it is existent. That makes god nothing special.

wrong, you are not paying attention ,What is being said is that there are only two possibles regarding God's modal status, either necessary or impossible. In other words no middle ground, if God exists he must exist he can't be a maybe, he could not have failed to exist, if there is a God there had to be a God. The only alternative is that if God does not exist it's impossible that God could have existed God either exists and it is necessary that he does or he doesn't exist and if so it's because he could not exit.

If you mean some notion of alternative possibilities that makes god special necessarily then no, one can speculate that something gave rise to god, god's god, but maybe god's god died, though previously greater than god, but now dead, so now god exists.

Nope doesn't work that way. God has to be eternal or he can't be at all., he could not have a cause.If God exists he exists as a necessity, A necessity doesn't have a cause,if it did it would be contingent.

The speculative alternative formulations are unbounded, hence the assertion of necessity is false.

wrong modal operators are not "unbounded." Yes there is a limitless field of speculation concerning God but NOT where modal operators are concerned.

Fail from the git go, but then, you did not fully define your terms so you might think you have some definitional alternatives to these failings.

Joseph Hinman (Metacrock) said...

Let's notice guys Stardust has tried to shift burden to me to prove things about the argument, but he has not demonstrated any problem, he's trying to fiddle with preliminaries. That's silly because it's such a famous argument it has a huge body of commentary as voluminous as the debates about evolution so there's no way to avoid the conclusion that Startdust doesn't know anything about the issue. That pretty much calls into questioning his groundless view that there are no valid arguments for God.


He's still confusing valid with proved. The point is valid refers to formally corrected of the presentation not the truth of the argument (which is it's "soundness").to show that it is not valid he has to show that there is a fallacy in the argument itself.

Just saying that speculation is wide o[en is not the demonstration of a fallacy it's an opinion.I think one will find that most logicians (or at least many) consider the argument valid but the issue they take with it is with its soundness.

B. Prokop said...

Joe,

That is one of the smartest things you've ever posted. I don't always agree with you, but this time you nailed it!

B. Prokop said...

I was referring to the 7:46 PM posting.

bmiller said...

@Joe Hinman,

I sort of agree with Bob. You post plenty of smart things, but you took the time to more thoroughly explain the argument. Thanks.

StardustyPsyche said...

bmiller said...

" This is what I want you to clear up with me regarding your position. Do you think how we can know how reality works? Is the universe intelligible?"
Ultimately with certainty? I doubt it very much. For example, physics is obviously incomplete. It is possible it will remain incomplete for as long as our species survives. There may very well be fundamental limits to our ability to understand and observe the very small and the very large.

But that does not make the universe completely unintelligible. We can make a very great deal of sense of the universe using our senses and reasoning.

" Of course mathematics is very useful in our investigation of the physical world. But the abstract science of mathematics is not the same as the science of the physical world. Differential equations allow infinite series to exist, which I know, sends you up the wall since you fume at “actual infinities” of any sort. It seems odd that you would appeal now to the concept of infinite series as the basis of reality."
I used the example of a matrix of differential expressions to communicate a matrix of cause and effect. I did not say an infinite series has a physical realization.

Much of mathematics is highly descriptive of the universe but not all mathematics has a physical realization. We can build a mathematical model of physics without that model being perfectly accurate, yet the model has demonstrated utility.


January 14, 2017 6:04 PM

Joseph Hinman (Metacrock) said...

is "Bob" Prokop? thanks both of you ;-)

StardustyPsyche said...

Joe Hinman said...
SP I will just start at the beginning
"IF God exists THEN it is logically necessary that(God exists)"
Only in the tautological sense that this statement applies to all existent things. If a thing exists then it is necessary that it exists since it is existent. That makes god nothing special."

" wrong, you are not paying attention ,"
Ha Ha Ha, you crack me up...

"What is being said is that there are only two possibles regarding God's modal status, either necessary or impossible. "
Which can be said of any existent thing, since it is here. The probability of its existence is 1.

"In other words no middle ground, if God exists he must exist he can't be a maybe, he could not have failed to exist, if there is a God there had to be a God. The only alternative is that if God does not exist it's impossible that God could have existed God either exists and it is necessary that he does or he doesn't exist and if so it's because he could not exit."
"Possible"? What do you mean by that word?

For example, is it possible that the universe has 57 dimensions? Does a speculated alternative automatically have the status of a possibility? What if their is some fundamental limit to reality making 57 dimensional space impossible? How can we know if something is possible or impossible?

Or do you assign a non-zero probability to all utterable speculations?


" Nope doesn't work that way. God has to be eternal or he can't be at all.,"
Why? Maybe today's god was created by a previous god, god's god, ad infinitum in an unbounded regression of god's.

Maybe a god can die. Maybe a god can be infinite in the past but finite in the future. Have you ever set up an integral from negative infinity to zero?

Maybe some infinite natural substance gave rise to god and our poor little brains just are not good enough to conceive of an infinitely existing substance.

Sorry Joe, "doesn't work that way" is a particularly weak argument. You will need to define god and justify that definition before we can even get past line 1.


January 14, 2017 7:46 PM

bmiller said...

@Stardusty Psyche,

Me:" This is what I want you to clear up with me regarding your position. Do you think how we can know how reality works? Is the universe intelligible?"
SP:”Ultimately with certainty? I doubt it very much. For example, physics is obviously incomplete. It is possible it will remain incomplete for as long as our species survives. There may very well be fundamental limits to our ability to understand and observe the very small and the very large.

But that does not make the universe completely unintelligible. We can make a very great deal of sense of the universe using our senses and reasoning.”

Let me ask this in a rather jarring way. “Are you saying the universe is chaos and not chaos?”

If the universe is chaotic and not intelligible, then when you think you are doing *science* you are actually chasing your tail so why bother. If it is not chaos and intelligible then we have a chance in principle to understand it, right? What do you think?

StardustyPsyche said...


Blogger Joe Hinman said...

" Let's notice guys Stardust has tried to shift burden to me to prove things about the argument, but he has not demonstrated any problem, he's trying to fiddle with preliminaries. That's silly because it's such a famous argument it has a huge body of commentary as voluminous as the debates about evolution so there's no way to avoid the conclusion that Startdust doesn't know anything about the issue. That pretty much calls into questioning his groundless view that there are no valid arguments for God.


He's still confusing valid with proved. The point is valid refers to formally corrected of the presentation not the truth of the argument (which is it's "soundness").to show that it is not valid he has to show that there is a fallacy in the argument itself."

Reading skill Joe.

"Joe Hinman said...

Stardust:

"There are no sound arguments for a theistic god as a necessary being that have ever been published into general circulation."


January 13, 2017 11:47 PM"



" Just saying that speculation is wide o[en is not the demonstration of a fallacy it's an opinion.I think one will find that most logicians (or at least many) consider the argument valid but the issue they take with it is with its soundness."

Indeed, the argument is unsound, which is what I said, and you quoted me on that January 13, 2017 11:47 PM

Reading skills Joe, reading skills.


January 14, 2017 7:58 PM

bmiller said...

@Stardusty Psyche,

BTW: "Will I get an answer to our exchanges on 1) and 2)?
I really want to understand your point of view on these 2."

I hope you haven't ignored this question.

StardustyPsyche said...

bmiller said...

" Let me ask this in a rather jarring way. “Are you saying the universe is chaos and not chaos?”
A chaotic system is a deterministic system that is too complicated to be analyzed in closed form. In principle a chaotic system can be precisely defined and predicted, but because there are an impracticably large number of variables we must resort to other analytical methods to get useful work done.

We have a postulate of the principle of non-contradiction most of us accept, as do I. Logically, if a proposition is true its negation is false.

In a logical expression x and not x is false, irrespective of x being true or false.


" If the universe is chaotic and not intelligible, "
Chaotic and intelligible are not wholly mutually exclusive. A chaotic system can be at least in part intelligible.


"then when you think you are doing *science* you are actually chasing your tail so why bother. "
Because it works. Science works. If you disagree then I suggest you throw away your cell phone, refuse your next MRI, place your computer in the rubbish, start muttering about fake moon landings, and go scrape out a stone technology hunter gatherer existence out is some wilderness someplace.


"If it is not chaos and intelligible then we have a chance in principle to understand it, right? What do you think?"
I think the quest for ever greater understanding of reality is truly thrilling. I am fully aware I will not achieve a complete understanding of our existence before I die, but I intend to get as far along as I am able.


January 14, 2017 9:04 PM

Ilíon said...

^ According to the 'Science!' fetishists, the universe *is* chaotic ... and (so the assertion runs) 'Science!' has demonstrated this.

Ilíon said...

... and further, 'Science!' fetishists assert that to deny either of their assertion that the universe is chaotic or that 'Science!' has demonstrated this -- that event happen without cause -- is to be "forever closed to some of the major findings on the rules that govern the Universe."

Joseph Hinman (Metacrock) said...

Stardust:

"There are no sound arguments for a theistic god as a necessary being that have ever been published into general circulation."

I have 52. 10 on RAP and 42 on Doxa (yes tried to make 43 as an obsession for HGTTG

ok now let's see you disprove the 200 studies that back my religious experience arguments.

StardustyPsyche said...

Joe Hinman said...

" Stardust:

"There are no sound arguments for a theistic god as a necessary being that have ever been published into general circulation."

I have 52. 10 on RAP and 42 on Doxa (yes tried to make 43 as an obsession for HGTTG"
Ok, so you acknowledge that your reading skills are preposterously lacking.



" ok now let's see you disprove the 200 studies that back my religious experience arguments."
Ha Ha Ha. You have never presented a sound argument for a theistic god here. Your above attempt failed miserably so now you try to cite some mythological 200.

Howzabout you present just 1? Right here, right now. Tick tock.


January 14, 2017 11:42 PM

Joseph Hinman (Metacrock) said...

in thye words of the great John Loftus buy the book

Joseph Hinman (Metacrock) said...

hey Dusty here is an article i wrote for an academic conference that's a pretty good summary of he major points of the book, read the the article,I know you can barely to read one page of anything pro God; it will make you feel angry to read it but you have to if you are ever to understand why you are so very wrong,

B. Prokop said...

In the midst of all this back and forth about the meaning of necessary and contingent, of chaotic and intelligible, about sound argument and invincible ignorance, we all (all of us, believer and atheist alike) would profit by spending some time in quiet, unhurried contemplation of what lies at the very heart of Christian doctrine - the Holy Trinity. Don't just glance over the doctrine - take a good ten minutes or so (which can be quite a long time, just try it!) with no distractions, no going-in agenda, no preconceptions. A visual aid, such as Rublev's Troitsa, could help. You'll be amazed at what might be revealed to you.

The implications and ramifications of said doctrine are vast - infinite, in fact. It annihilates questions such as "Who made God?", and throws all concern about infinite regression and probability into the dumpster.

The doctrine of the Trinity tells us that the core, the fundamental bedrock truth about reality, about being itself, is Love, community, relationship.

The Father is the great "I am" (as revealed to Moses). The Father eternally expresses Himself, and this self-expression, the Word of God, is the Son. The Father and the Son behold each other, and love what they see. This mutual Love is Himself a Person, the Holy Spirit, eternally proceeding from the Father and the Son. And the Three are in such perfect union that they, while remaining three distinct Persons, are nevertheless One God.

And that, Dear Internet Friends, is the truest thing you'll ever read.

And that is my challenge to you - just 10 minutes of your time.

StardustyPsyche said...

Joe Hinman said...

" in thye words of the great John Loftus buy the book"
Fail.

January 15, 2017 6:19 AM

Walter said...

And that is my challenge to you - just 10 minutes of your time.

I have contemplated the Trinity for far longer than ten minutes, and found that I could not believe it to be true. It is one the main reasons that, although I believe in God, I cannot affirm orthodox Christianity.

StardustyPsyche said...

B. Prokop said...

" In the midst of all this back and forth about the meaning of necessary and contingent, of chaotic and intelligible, about sound argument and invincible ignorance, we all (all of us, believer and atheist alike) would profit by spending some time in quiet, unhurried contemplation of what lies at the very heart of Christian doctrine - the Holy Trinity. Don't just glance over the doctrine - take a good ten minutes or so (which can be quite a long time, just try it!) with no distractions, no going-in agenda, no preconceptions. A visual aid, such as Rublev's Troitsa, could help. You'll be amazed at what might be revealed to you."
I get the feeling you sincerely feel you are doing a public service here.


" The implications and ramifications of said doctrine are vast - infinite, in fact. It annihilates questions such as "Who made God?", and throws all concern about infinite regression and probability into the dumpster."
Wrong, the trinity solves none of those problems.

" The doctrine of the Trinity tells us that the core, the fundamental bedrock truth about reality, about being itself, is Love, community, relationship."
Being is feelings in your view. This is a highly anthropocentric view, typical of Christians who commonly express the idea that the universe was made for us. Actual observation of the universe shows humankind as a tiny speck, microscopic by comparison.


" The Father is the great "I am" (as revealed to Moses). The Father eternally expresses Himself, and this self-expression, the Word of God, is the Son. The Father and the Son behold each other, and love what they see. This mutual Love is Himself a Person, the Holy Spirit, eternally proceeding from the Father and the Son. And the Three are in such perfect union that they, while remaining three distinct Persons, are nevertheless One God."
Ancient mythology.

" And that, Dear Internet Friends, is the truest thing you'll ever read.

And that is my challenge to you - just 10 minutes of your time."
Sorry, you sound sort of intoxicated by all your stories, wozzy and wanting to share the high by passing it around. I prefer to stay sober.


January 15, 2017 7:12 AM

Joseph Hinman (Metacrock) said...

And that is my challenge to you - just 10 minutes of your time.

I have contemplated the Trinity for far longer than ten minutes, and found that I could not believe it to be true. It is one the main reasons that, although I believe in God, I cannot affirm orthodox Christianity.

January 15, 2017 7:56 AM


no offense but could you tell me exactly what you think the doctrine says?

StardustyPsyche said...

Joe Hinman said...

" hey Dusty here is an article i wrote for an academic conference that's a pretty good summary of he major points of the book,"
Not Found
Error 404

You have difficulties reading, linking, and reasoning. If you have a sound argument for god it should be very easy to paste it here. All your dodging makes you look dishonest.


January 15, 2017 6:25 AM

Joseph Hinman (Metacrock) said...

Dusty I know you will never bother to read one word a out any God argument that';s not mocking and ridiculing from an atheist,you don't want truth, you don't want answers,

StardustyPsyche said...

Walter said...

" And that is my challenge to you - just 10 minutes of your time.

I have contemplated the Trinity for far longer than ten minutes, and found that I could not believe it to be true. It is one the main reasons that, although I believe in God, I cannot affirm orthodox Christianity."
Did you find some particular rational arguments for god persuasive or is your belief based on personal experience?


January 15, 2017 7:56 AM

bmiller said...

@Stardusty Psyche,

BTW: "Will I get an answer to our exchanges on 1) and 2)?
I really want to understand your point of view on these 2."

I hope you haven't ignored this question.

I am using the term “chaos” in the sense of complete and total lack of order. It seems to me that you took I meant it in term of “chaos theory” which is a totally different concept. It’s my own pet peeve that Robert L. Devaney chose to use the term “chaos” for this theory.

Can you please respond again with the definition of “chaos” in the sense I intended?

StardustyPsyche said...

Joe Hinman said...

" Dusty I know you will never bother to read one word a out any God argument that';s not mocking and ridiculing from an atheist,you don't want truth, you don't want answers,"
Dodge, you went off about my supposed denial of the validity of your silly argument after you had quoted my word as "sound" in the first place.

You then claim your sound arguments total "I have 52. 10 on RAP and 42 on Doxa" with some 200 studies to back them up yet you are unable to paste a single one here.

Then you posted a broken link.

So far on this thread you have been fail after fail.

If you have 52 sound arguments for god it should be a simple matter to paste here just 1.


January 15, 2017 8:09 AM

StardustyPsyche said...

bmiller said...
" @Stardusty Psyche,
BTW: "Will I get an answer to our exchanges on 1) and 2)?
I really want to understand your point of view on these 2."
I hope you haven't ignored this question."
Not ignoring, I thought I had pretty much discussed cause and effect. In my view the problem of first cause remains unsolved but to consider it we need to view cause and effect as a mutual transitory temporal process, not assign a static title of "cause" to one macro object and "effect" to another macro object.

" I am using the term “chaos” in the sense of complete and total lack of order. "
I don't know what that would be in our observable universe.

"It seems to me that you took I meant it in term of “chaos theory” which is a totally different concept. It’s my own pet peeve that Robert L. Devaney chose to use the term “chaos” for this theory."
Ok, well you are gonna have to take that up with Bob!

" Can you please respond again with the definition of “chaos” in the sense I intended?"
I know of no place in our observable universe that is in a state of complete and total lack of order. You seem to be considering some sort of amorphous soup or perhaps some perfectly uniform substrate of something...don't know where you are going with this, but quite apparently our observable universe is highly ordered.


January 15, 2017 8:21 AM

Ilíon said...

B.Prokop: "The Father is the great "I am" (as revealed to Moses)."

That's what I always thought, too. But, in fact, "the God of the Old Testament" is the Second Person, the Son, not the First, the Father.

The New Testament writers make this clear, in *so* many passages, if only we would set our preconceptions aside and *pay attention* to what both they and the OT writers actually wrote.

For instance --
Who does the OT name as the Creator? Elohim/YHWH (I AM; aka Jehovah). Who does the OT name as the Creator? The Son, the Second Person.

For instance --
What Divine Name does the Christ attribute to himself (and which is why the Jews who rejected his claim *had* to kill him)? I AM.

It is I AM who revealed himself to Moses and led the Israelites out of Egypt. It is the Christ who is I AM.

Ilíon said...

B.Prokop: "The doctrine of the Trinity tells us that the core, the fundamental bedrock truth about reality, about being itself, is Love, community, relationship."

Walter: "I have contemplated the Trinity for far longer than ten minutes, and found that I could not believe it to be true. It is one the main reasons that, although I believe in God, I cannot affirm orthodox Christianity."

Walter, nothing about reality makes sense -- it does not cohere -- except through the reality of the Trinity.

Take morality, for example, and the silly 'Euthyphro Dilemma' -- if God is not a multiplicity of Persons, then the 'Euthyphro Dilemma' isn't so silly, and really is a dilemma ... and either horn of it forces us to deny at least one thing we *know* to be true of morality. But, if God is a multiplicity of Persons, then the 'Euthyphro Dilemma' is a pseudo-dilemma, it has no power, and the coherence of morality is preserved.

Ilíon said...

bmiller: "I am using the term “chaos” in the sense of complete and total lack of order."

A better way to understand 'chaos' is as a disconnect between cause and effect. That is, if 'chaos' were a fact, then events could and would occur without cause.

B. Prokop said...

"if God is not a multiplicity of Persons, then the 'Euthyphro Dilemma' isn't so silly, and really is a dilemma"

Exactly so. Also, if God is not a Trinity, then the statement "God is Love" (which is not from John Lennon, but rather from John the Evangelist) makes Him a monster. It's ultimately why Islam is such a cruel and inhuman religion.

Ilíon said...

^ I'm sure B.Prokop understands this, but others may not --

I do affirm the Trinity, as all actual Christians do.

BUT, the reason I am speaking of God as being "a multiplicity of Persons" rather than a Trinity is because I am not speaking or arguing of what we can know about God from the basis of the Christian revelation, but rather of what we can know about God from the basis of reason and experience.

The astute reader will notice that the only time I speak/argue from the basis of the Christian revelation is when that is the topic.

Walter said...

http://trinities.org/dale/SinglePerfect.pdf

Bob,

The link above will take you to a paper written by an analytic philosopher and author of the Stanford Encyclopedia of Philosophy entry on the Trinity. In this paper he argues that a perfect being who does not experience peer-love is not a contradictory concept, while the many-selves god of social trinitarianism might very well be incoherent.

SteveK said...

Dusty
"Interesting. All attempts to prove the existence of god that I have ever read or heard fail immediately."

I rarely call someone a liar.....You are a liar. You agreed that a proof stated in the 'Aswedenism' post was legitimate.

Ilíon said...

Walter: "In this paper he argues that a perfect being who does not experience peer-love is not a contradictory concept ..."

Who said it was? The claim is that *every (important) thing else* that we know to be true becomes incoherent if God is "a perfect being who does not experience peer-love".

Walter: "In this paper he argues that ... the many-selves god of social trinitarianism might very well be incoherent."

Translation: the author cannot rationally establish the assertion he desires to make that "social trinitarianism" *is* incoherent ... but he's still gonna make it anyway, just in Academese rather than in English.

B. Prokop said...

"a perfect being who does not experience peer-love is not a contradictory concept"

Nonsense. Next to "contradictory" in the dictionary is Dale Tuggy's paper. If God were a unitary being, then He would require some entity other than Himself in order to love. But love is part and parcel of perfection. If God were to require the existence of some other being in order to love, then He is not perfect.

Ergo, a Triune God is the only game in town. Otherwise, he is either not perfect, or He is in love with Himself (i.e., a monster).

Walter said...

or He is in love with Himself (i.e., a monster).

If God is a perfect being then it is neither monstrous nor narcissistic to love that which is perfect. You have said nothing so far to rebut Tuggy's argument, you are simply doubling down on your assertions.

bmiller said...

@Stardusty Psyche,

Our exchange:
“BTW: "Will I get an answer to our exchanges on 1) and 2)?
I really want to understand your point of view on these 2."
I hope you haven't ignored this question."
Not ignoring, I thought I had pretty much discussed cause and effect. In my view the problem of first cause remains unsolved but to consider it we need to view cause and effect as a mutual transitory temporal process, not assign a static title of "cause" to one macro object and "effect" to another macro object.”

I would have preferred that you succinctly explained your concept of cause and effect that covered these questions:. Do they exist? Are they illusions? Can we cause effects?

I still can’t tell. On the one hand you insist that the concepts make no sense, but on the other you say that perhaps they are illusions and yet you seem to allow that we can actually cause effects although without 100% certainty. You promised to un-muddle the simplistic thinking on this topic. You have not achieved that goal.

Let me highlight some problems with your concepts as I see them.

If everything is causing everything else, then you also are part of that interaction yourself and therefore everything you think you are doing for any particular reason is not actually being done for that particular reason. This includes any reasons you bring forth as arguments. For everything in the universe is causing everything you are doing, not *you*.

Science uses experimentation as part of the process to gain knowledge. Experiments proceed from known initial states, cause something to change and observe the effects. If the experimenter is merely part of the universal process of interacting causes, then he is not really responsible for any changes and experimentation is meaningless in reality. In this case science itself is an illusion.

If one concludes that science is an illusion then it is silly to use it as a premise for any argument. In fact other non-Christian cultures actually hold this.

B. Prokop said...

"If God is a perfect being then it is neither monstrous nor narcissistic to love that which is perfect."

The very definition of narcissism is to be in love with one's self. In fact, you can't really use the word "love" to describe such a thing. It is the complete opposite of the self-giving which is the essence and fundamental attribute of love.

StardustyPsyche said...

SteveK said...

" Dusty
"Interesting. All attempts to prove the existence of god that I have ever read or heard fail immediately."

I rarely call someone a liar.....You are a liar. You agreed that a proof stated in the 'Aswedenism' post was legitimate."
Really? You claim I found an asserted proof of the existence of god to be "legitimate" in the sense that it does not fail.

You are either mistaken or dishonest.

There has never been a sound argument for the existence of god published into general circulation, on this site, or anyplace else.

Care to provide the link to this alleged statement of "legitimate"? Else your comment is mere chaff.


January 15, 2017 11:38 AM

Walter said...

The very definition of narcissism is to be in love with one's self

Narcissism has negative connotations when human beings engage in it. God is not a human being.

Another reason that I do not believe in a many-selves god is that it appears to fly in the face of divine simplicity. Why three and not ten or an infinity of persons within God? The idea of an intelligent, unipersonal first cause seems far more elegant in its simplicity.

Joseph Hinman (Metacrock) said...

There has never been a sound argument for the existence of god published into general circulation, on this site, or anyplace else.

there has never been an intelligent word uttered by an atheist. dusty you ha no answeror the 200 studies, you can't answer my God arguments,I've kicked your ass every time you come over there on my blog or CADRE and spout your ignorant nonsense.

you don't have an answer for the modal argument, you don't have an answer for the experience arguments, you don't have an answer for the ground of being arguments, you have nothing to say but groundless pronouncement that are backed by nothing.

Joseph Hinman (Metacrock) said...

Dusty says

Dodge, you went off about my supposed denial of the validity of your silly argument after you had quoted my word as "sound" in the first place.

no you merely wrong on both counts valid and sound, that's what i said in the appropriate times

You then claim your sound arguments total "I have 52. 10 on RAP and 42 on Doxa" with some 200 studies to back them up yet you are unable to paste a single one here.

the 200 back u the experience arguments and the others are backed by more studies,

Then you posted a broken link.

anyone can have a broken link and ot kj now it, which one?

So far on this thread you have been fail after fail.

hey man you showed your true ignorance on the modal argument you have failed to make a single argument on any other, all you are doing is calling arguments names then saying the word fail,that's not an an argument, you are merely gaining the evidence that shows your true ignorance,

If you have 52 sound arguments for god it should be a simple matter to paste here just 1.

Joseph Hinman (Metacrock) said...
This comment has been removed by the author.
Joseph Hinman (Metacrock) said...

9 God arguments om religious a priori

42 on Doxa

Joseph Hinman (Metacrock) said...

article summarizing my book

StardustyPsyche said...

bmiller said...

@Stardusty Psyche,

" I would have preferred that you succinctly explained your concept of cause and effect that covered these questions:."
I think I did, but I will make it short then...

" Do they exist?"
As a real mutual transitory temporal process, yes.

"Are they illusions?"
As most people conceive of them, yes. As a description of the progress of reality over time, no.

" Can we cause effects?"
In the sense that we were part of a process of mutual causes and effects in the past, in the present, and into the future, yes. Our personal experience of these processes is a highly simplified representation of the underlying elements affected.

" I still can’t tell. On the one hand you insist that the concepts make no sense, "
The notion of assigning the title of "cause" to an apparently static object is a gross oversimplification that leads to erroneous conclusions.

"but on the other you say that perhaps they are illusions and yet you seem to allow that we can actually cause effects although without 100% certainty. You promised to un-muddle the simplistic thinking on this topic. You have not achieved that goal."
Ok, well, there is a great deal written on the subject. You can search for cause and effect illusion and get some more ideas on the subject.

One common objection to the assignment of cause is that we must fix a point in time. Suppose a rolling billiard ball strikes another, and then that one strikes another and so on. Which ball is the cause and which ball is the effect? The choice is arbitrary. In truth all the balls were both cause and effect to each other in a temporal sequence.


" Let me highlight some problems with your concepts as I see them.

If everything is causing everything else, then you also are part of that interaction yourself and therefore everything you think you are doing for any particular reason is not actually being done for that particular reason."
Humans suffer from self delusions and I am no exception to that condition.

"This includes any reasons you bring forth as arguments. For everything in the universe is causing everything you are doing, not *you*."
Who or what is this *you*? I am a distributed parallel process device.

" Science uses experimentation as part of the process to gain knowledge. Experiments proceed from known initial states, cause something to change and observe the effects. "
Ideally, but not perfectly in practice.

"If the experimenter is merely part of the universal process of interacting causes, then he is not really responsible for any changes and experimentation is meaningless in reality. "
Separating the influence of the experimenter from the results of the experiment is an important consideration. Hence, John Bell stood against “measurement”.

“In this case science itself is an illusion.”
Science is in part a means to separate the illusory from the real.

“ If one concludes that science is an illusion then it is silly to use it as a premise for any argument. “
If you think science is an illusion then your cell phone, MRI, TV, and computer are all illusions, so I suggest you just throw them away.

“In fact other non-Christian cultures actually hold this.”
Foolishly

January 15, 2017 12:09 PM

StardustyPsyche said...

Joe Hinman said...

" you don't have an answer for the modal argument, "
It is unsound, as I showed above.
x -> g
x
g

is a valid argument. So what? It is unsound.
x = "the sky is blue today"
g = "StardustyPsyche is god"
Don't contradict me the argument is valid!!!

"you don't have an answer for the experience arguments, you don't have an answer for the ground of being arguments, you have nothing to say but groundless pronouncement that are backed by nothing."
You have not presented any such arguments here thusfar.


January 15, 2017 1:06 PM

Joseph Hinman (Metacrock) said...

oe Hinman said...

" you don't have an answer for the modal argument, "
It is unsound, as I showed above.
x -> g
x
g

is a valid argument. So what? It is unsound.
x = "the sky is blue today"
g = "StardustyPsyche is god"
Don't contradict me the argument is valid!!!

"you don't have an answer for the experience arguments, you don't have an answer for the ground of being arguments, you have nothing to say but groundless pronouncement that are backed by nothing."
You have not presented any such arguments here thusfar.

you are just showing us that you are too stupid to follow a logical argument, The OA is logical if you follow the logic it makes sense, you respond by assortment of meaningless satmetms. That is mockery, as SK says hye who mocks others moks hismef,

SteveK said...
This comment has been removed by the author.
SteveK said...

Dusty,
You are a liar. A sloppy thinker that I took advantage of, as the following exchange aptly demonstrates, but a liar nonetheless. Enjoy.

--------
You: "If you define god as [Dusty defines god] and we observe a being who [satisfies that definition] then that is evidence for that limited formulation of god."

Me: [defining another god] "...you tell me if this is a legitimate setup"
You: [repeating my definition and setup] "By definition that is literally correct."

Me: "Since that setup is correct, we have scientific evidence for god and atheism is not a rational position to hold."
You: "Sure..."

Me: "The setup is legitimate, you said. Don't ever say there's no scientific evidence for god."
You: "Which god?"

Me: "The one identified in the set up you said was legitimate."

https://www.blogger.com/comment.g?postID=5146759188289684594&blogID=10584495&isPopup=false&page=2&bpli=1


[commentary: Dusty claims to observe a being that he refers to as god. Is he right? How does Dusty know that this being is god? Maybe Dusty has an incorrect definition. Maybe god doesn't exist at all. Dusty is trying to prove that god exists, but instead he is defining god into existence and then going out to look for a match.

Dusty thinks science can discover the ontology of god through experiment, but the ontology is a product of Dusty's mind, not a product of scientific discovery. I was happy to exploit his sloppy thinking since Dusty insists he's so smart.]

StardustyPsyche said...

Joe Hinman said...

" you are just showing us that you are too stupid to follow a logical argument, The OA is logical if you follow the logic it makes sense, "
So does this
x -> g
x
g
See, it makes sense that I am god because I have constructed a valid argument with that conclusion.

I can use the OA to prove I am god because I can conceive of myself as the greatest possible being. How absurd.

"you respond by assortment of meaningless satmetms. That is mockery,"
Indeed, mockery is just what nonsense like yours richly deserves.

" as SK says hye who mocks others moks hismef,"
How silly, he who mocks absurd ideas sheds light on reasonable inquiry.


January 15, 2017 2:50 PM

Kevin said...

"There has never been a sound argument for the existence of god published into general circulation, on this site, or anyplace else."

How do you know this?

StardustyPsyche said...

I rarely call someone a liar.....You are a liar. You agreed that a proof stated in the 'Aswedenism' post was legitimate."

" SteveK said...

Dusty,
You are a liar. A sloppy thinker that I took advantage of, as the following exchange aptly demonstrates, but a liar nonetheless. Enjoy.

--------
You: "If you define god as [Dusty defines god] and we observe a being who [satisfies that definition] then that is evidence for that limited formulation of god."

Me: [defining another god] "...you tell me if this is a legitimate setup"
You: [repeating my definition and setup] "By definition that is literally correct."

Me: "Since that setup is correct, we have scientific evidence for god and atheism is not a rational position to hold."
You: "Sure..."

Me: "The setup is legitimate, you said. Don't ever say there's no scientific evidence for god."
You: "Which god?"

Me: "The one identified in the set up you said was legitimate."

https://www.blogger.com/comment.g?postID=5146759188289684594&blogID=10584495&isPopup=false&page=2&bpli=1"

That isn't a sound argument for the existence of god, that is just a hypothetical observation of a particular being doing particular things.


" [commentary: Dusty claims to observe a being that he refers to as god. Is he right?"
You can rename a rock to be called god, demonstrate the godrock exists, and claim to have given a sound argument proving the existence of god. If that somehow makes you feel good, well, whatever.


" How does Dusty know that this being is god? Maybe Dusty has an incorrect definition. Maybe god doesn't exist at all. Dusty is trying to prove that god exists, "
Huh?

"but instead he is defining god into existence and then going out to look for a match."
???

" Dusty thinks science can discover the ontology of god through experiment,"
Lay off the sauce, my friend, you now blithering and blathering.

" but the ontology is a product of Dusty's mind, not a product of scientific discovery. I was happy to exploit his sloppy thinking since Dusty insists he's so smart.]"
Your thinking is scattered in bits and pieces of phrases and snipped quotes.

I suggest you clear your mind and try to think rationally from some reasonable starting point.


January 15, 2017 3:45 PM

StardustyPsyche said...

Legion of Logic said...

"There has never been a sound argument for the existence of god published into general circulation, on this site, or anyplace else."

" How do you know this?"
Fair question.

Note I did not make the strong claim that x has never been written down, or x has never been thought of, or x has never been spoken. Proving that would require proving the negative by probing the minds and writings and words of all human beings, which I am obviously incapable of doing.

The key qualifier that makes my claim defensible is "general circulation". When a major intellectual achievement gets published into general circulation it gets recognized and becomes part of reference documents and major presentations.

Lots of people have tried to prove the existence of god for millennia. Minor and weak arguments could possibly be published without notice. But a sound argument for god would be so unique (being the first of its kind), and such a major event it would be sure to be picked up and become universally known to all major sources in the field.

Everybody interested knows the Kalam, the ontological, the argument from morality, and a few others. They all fail. If some new argument came along that was for once a sound argument the processes of detecting and disseminating this argument would surely make it known to us all who are interested, providing the argument got published into general circulation.


January 15, 2017 4:12 PM

StardustyPsyche said...


Joe,
I started looking at your links. Wow, what a shitstorm of nonsense. It will take time to go through, although most of them are just retreads of long ago debunked foolishness here is one sample of your silliness that clearly deserves the description of "fails immediately".
http://religiousapriori.blogspot.com/2009/01/god-on-brain-argument-from-innate-god.html


"Science doesn't recognize innate ideas. "
Wrong. Noam Chomsky and many others.


"Many people might think it does, they are fooled into thinking by the nature of genetic disposition. Genes determine many things in our behavior and of course everything about our physical endowment. Those are not ideas. "
Brain structure gives rise to ideas. DNA drives brain structure. DNA calls out the structure of the hardwiring. When the hardwiring functions we all end up having similar thoughts because our brains are constructed similarly because our DNA is similar.

Joe fails again right out of the gate.

This fundamental failure of Joe ties in to several of his other failures involving mystical experiences and other brain related "arguments".

Kevin said...

"But a sound argument for god would be so unique (being the first of its kind), and such a major event it would be sure to be picked up and become universally known to all major sources in the field."

This seems to give an argument's critics more authority than its proponents, as though the critics are more knowledgeable or objective. I don't believe the truth of an idea that isn't blatantly obvious is necessarily linked to a tendency toward universal acceptance - the majority can be, and is often, wrong.

B. Prokop said...

The major problem here is that Stardusty believes himself to be the final arbiter in the matter of whether any particular argument "fails" - i.e., if it doesn't convince him, then it is obviously unsound.

The facts speak otherwise. The Five Ways, the Kalam argument, the argument from morality, the argument from reason, the argument from there being something rather than nothing, the argument from the reliability of the Gospel narratives, the argument from personal experience... all are sound arguments that have convinced quite literally billions of people over the centuries. In what way can they be labeled failures after so much undeniable success?

Or perhaps Stardusty has a very different definition of the word success - something like "It has to convince me!"

StardustyPsyche said...

Legion of Logic said...

"But a sound argument for god would be so unique (being the first of its kind), and such a major event it would be sure to be picked up and become universally known to all major sources in the field."

" This seems to give an argument's critics more authority than its proponents, as though the critics are more knowledgeable or objective. I don't believe the truth of an idea that isn't blatantly obvious is necessarily linked to a tendency toward universal acceptance - the majority can be, and is often, wrong."
True, but not my assertion.

Any sound argument in general circulation would make it to the list of apologists and be placed at the top by them, even if others did not universally accept it.

I am starting to poke around at
http://religiousapriori.blogspot.com
That Joe linked to me. Lots of arguments there, all of them unsound. If some new argument came out, the first sound argument for god, you can be sure that sites such as this would pick it up and post it, and folks like Joe would lead off with it.

In fact, the apologists always lead off with the same old fallacious nonsense.


January 15, 2017 5:36 PM

StardustyPsyche said...

B. Prokop said...

" The major problem here is that Stardusty believes himself to be the final arbiter in the matter of whether any particular argument "fails" - i.e., if it doesn't convince him, then it is obviously unsound."
Wrong, I argue on the merits, not from personal incredulity. I may also be personally incredulous, but that is not the reason the arguments fail.

" The facts speak otherwise. The Five Ways,"
Ha Ha Ha, Aquinas...tee hee hee.

" the Kalam argument,"
Special pleading, non-sequtur, false dichotomy.

" the argument from morality, "
Confusion between near universality of sensibilities versus an outside arbiter of absolute morality.

"the argument from reason, "
Pure ignorance.

"the argument from there being something rather than nothing,"
Special pleading, non-sequitur, false dichotomy

" the argument from the reliability of the Gospel narratives, "
Historical nonsense.

"the argument from personal experience... "
Confusion between fantasy and reality

"all are sound arguments"
Preposterous. You have no clue about the glaring defects of these arguments.


" that have convinced quite literally billions of people over the centuries."
Indeed, look how gullible people are, about 60 million even voted for Trump. Sad.


" In what way can they be labeled failures after so much undeniable success?"
They are logical failures, rational failures.


January 15, 2017 6:14 PM

SteveK said...

"That isn't a sound argument for the existence of god, that is just a hypothetical observation of a particular being doing particular things."

That's not what you said. You said it was legitimate. Liar!

SteveK said...

Liars can be redeemed though. Tell me, Liar, why the argument isn't sound.

StardustyPsyche said...

SteveK said...

" Liars can be redeemed though. Tell me, Liar, why the argument isn't sound."
Asked and answered. Let me know when you have a serious argument to make.


January 15, 2017 7:06 PM

StardustyPsyche said...


Closer Hartshorne's version

1. God is either necessary or impossible.
2. God can be conceived without contradiction.
3. Whatever can be conceived without contradiction is not impossible.
4. God is not impossible.
5. God is necessary.
6. If God is necessary, then God exists.
7. God exists.
http://religiousapriori.blogspot.com/2010/08/modal-argument.html

Line 3 is a non-sequitur. Just because we can imagine something that does not contradict itself as we imagine it does not mean the reality of the universe can possibly accommodate a realization of that fantasy.

Hartshorne is asserting that mere fantasy is sufficient to allow for external realization. He obviously has a hard time separating fantasy from reality, but that is typical of the theistic mind.

The whole argument hinges on thinking makes it so, an absurd notion. Why anybody takes this nonsense at all seriously is truly a wonderment for me.

And so it goes with everything Joe puts out there, nonsense upon nonsense.

Kevin said...

"Wrong, I argue on the merits, not from personal incredulity."

How do you know you are the more qualified to recognize an argument's merits?

Joseph Hinman (Metacrock) said...

Closer Hartshorne's version

1. God is either necessary or impossible.
2. God can be conceived without contradiction.
3. Whatever can be conceived without contradiction is not impossible.
4. God is not impossible.
5. God is necessary.
6. If God is necessary, then God exists.
7. God exists.
http://religiousapriori.blogspot.com/2010/08/modal-argument.html

Line 3 is a non-sequitur. Just because we can imagine something that does not contradict itself as we imagine it does not mean the reality of the universe can possibly accommodate a realization of that fantasy.

p3 is the lynch pin of the thing, it's anything but irrelevant, the argent turns upon it,

Hartshorne is asserting that mere fantasy is sufficient to allow for external realization. He obviously has a hard time separating fantasy from reality, but that is typical of the theistic mind.


that;s stupid, no logician in the world thinks that, he is not saying that,

The whole argument hinges on thinking makes it so, an absurd notion. Why anybody takes this nonsense at all seriously is truly a wonderment for me.

so very ignorant, it's own thread so I am going to put it up on CADRE blog for tomorrow. please check deserves it's own thread, I'll link over here.

bmiller said...

@Stardusty Psyche,

Me:"If everything is causing everything else, then you also are part of that interaction yourself and therefore everything you think you are doing for any particular reason is not actually being done for that particular reason."
SP:"Humans suffer from self delusions and I am no exception to that condition."

Your self-identification as being deluded is noted.

This exchange:
Me:" Science uses experimentation as part of the process to gain knowledge. Experiments proceed from known initial states, cause something to change and observe the effects. "
SP:”Ideally, but not perfectly in practice.”
Me:"If the experimenter is merely part of the universal process of interacting causes, then he is not really responsible for any changes and experimentation is meaningless in reality. "
SP:"Separating the influence of the experimenter from the results of the experiment is an important consideration. Hence, John Bell stood against “measurement”."

Me:“In this case science itself is an illusion.”
SP:"Science is in part a means to separate the illusory from the real."

If you claim that everything is caused by everything and nothing in particular, then experiments also are caused by nothing in particular making science nonsense. Your claims make nonsense of science.

Me:“ If one concludes that science is an illusion then it is silly to use it as a premise for any argument. “
SP:"If you think science is an illusion then your cell phone, MRI, TV, and computer are all illusions, so I suggest you just throw them away."

Thank you for making my point for me. Your imaginary universe is nonsense.

StardustyPsyche said...

Legion of Logic said...

" "Wrong, I argue on the merits, not from personal incredulity."

How do you know you are the more qualified to recognize an argument's merits?"
That's what rational engagement is for, else it would merely be argument from authority.

In science there are no authorities. We might give a recognized scientist more attention, but the argument must stand on its merits, not the asserted qualifications of the individual.


January 15, 2017 7:50 PM

Joseph Hinman (Metacrock) said...

It's so sophomoric to reduce the work of a recognized great thinker to "he thinks thinking makes it so."No he did not think that.He thought that the ontological principle is true. in other words if the terms of a proportionate spell out the truth content of the propositomn when understood then we have to assume the truth of the argument if the prepositions are valid.

Tillich's example of this principle is the principle of truth cannot be disputed without admitting to he validity of the principle. One can only say the principle is false if one is willing to admit that truth exists and this principle departs from it, Thus to dispute the truth of truth is to accept the proportion that truth exists. Truth can never be disputed as truth or as sound based upon a logical denial.

Heartshorne's version is the evocation of necessity with the possibility of god. In other words necessity is such that if X is necessary and possible the possibility of X means it must exist because it can't be merely possible if it's necessary, That's where no p3 comes in. The only two possibilities for go are necessary and impossible, If God is possible hes not impossible and thus must be necessary.

This is all guaranteed by the nature of modal operators, Modal operators are words that disclose the modes of being; hence"modal" logic. Modes of being are states such as necessity or contingnecy.

StardustyPsyche said...

3. Whatever can be conceived without contradiction is not impossible.

SP Line 3 is a non-sequitur. Just because we can imagine something that does not contradict itself as we imagine it does not mean the reality of the universe can possibly accommodate a realization of that fantasy.

"p3 is the lynch pin of the thing, it's anything but irrelevant, the argent turns upon it,"
I never said it was irrelevant, just mistaken.

SP Hartshorne is asserting that mere fantasy is sufficient to allow for external realization. He obviously has a hard time separating fantasy from reality, but that is typical of the theistic mind.


"that;s stupid,"
Indeed.

" no logician in the world thinks that, "
Hartshorne did, probably without realizing it as such.

"he is not saying that,"
He just did. That's what the words say, and I agree, it is stupid.

SP The whole argument hinges on thinking makes it so, an absurd notion. Why anybody takes this nonsense at all seriously is truly a wonderment for me.

"so very ignorant,"
Indeed.

" it's own thread so I am going to put it up on CADRE blog for tomorrow. please check deserves it's own thread, I'll link over here."
Maybe, you are much more of a jerk on your own blog. You are one of those moderators that acts like a real ass on your own turf. Plus your disregard for the use of spell checking is somewhat burdensome to read. At least here you take the time to correct enough of your mistakes that the remaining typos are easy to decipher.

When somebody will not bend to your fallacious statements and baseless accusations of dishonesty and trolling you ban them. If that gets you more participation from like minded people who enjoy an echo chamber, well, go for that approach then.


January 15, 2017 9:03 PM

SteveK said...

Dusty
Quoted and proven. Let us know when you'd like to stop lying about what you said.

StardustyPsyche said...

Joe Hinman said...

" It's so sophomoric to reduce the work of a recognized great thinker to "he thinks thinking makes it so."
That is what the ontological argument boils down to. It is a convoluted way to dress up that simple absurdity in sophisticated sounding language in an attempt to give a preposterous idea airs of legitimacy.


" Tillich's example of this principle is the principle of truth cannot be disputed without admitting to he validity of the principle. One can only say the principle is false if one is willing to admit that truth exists and this principle departs from it, "
In what sense does truth "exist"?
Logical arguments are founded on postulates that may be accepted provisionally or as articles of faith. We scientifically minded people accept them provisionally.


" Heartshorne's version is the evocation of necessity with the possibility of god. In other words necessity is such that if X is necessary and possible the possibility of X means it must exist because it can't be merely possible if it's necessary,"
Ok, you set up a definition...

" That's where no p3 comes in. The only two possibilities for go are necessary and impossible,"
Only if you define god that way. I can define god with other possibilities.

" If God is possible hes not impossible and thus must be necessary."
By definition, but that is not my objection to 3.

" This is all guaranteed by the nature of modal operators, Modal operators are words that disclose the modes of being; hence"modal" logic. Modes of being are states such as necessity or contingnecy."
Given your definitions of god and logic, that almost works, but defining god into existence is just a form of begging the question.

The flaw in 3. is the notion that imagining things in a certain way means they must be realizable in actual outside existence.

Premise 3. is simply unsound suffering from being a non-sequitur.

Merely because a thought can be conceived without contradiction it does not follow that it must be not impossible.

I can think of things that seem to be free of contradiction yet the reality of the universe may be that such a thought cannot be realized and is thus impossible.

Line 3. is unsound, and thus the ontological argument is unsound.


January 15, 2017 9:31 PM

Joseph Hinman (Metacrock) said...

I imagine you are going want to say this stuff in response to my CADRE article which I am writing now,so I ask you to can he responses stop talking about it unit I get the article up-.that will happen tonight,

Joseph Hinman (Metacrock) said...

The flaw in 3. is the notion that imagining things in a certain way means they must be realizable in actual outside existence.

Premise 3. is simply unsound suffering from being a non-sequitur.

not what it says

Merely because a thought can be conceived without contradiction it does not follow that it must be not impossible.

yes it does, Impossible in logo does not mean it doesn't exist or that it seems it wont work given the laws of physics, the world of logic is not ruled by empirical rules. It means one thing only, impossible in logic means that it is a contradiction in logic.If there is no contradiction it's not impossible.

I can think of things that seem to be free of contradiction yet the reality of the universe may be that such a thought cannot be realized and is thus impossible.

that's wrong for the reason i just said. it;s not abouit what works in the real world of physics it's logoc. only if it contradicts logically is it impossible.

let's re,e,bert this comare up in relation to validity and that a question of logic,


Line 3. is unsound, and thus the ontological argument is unsound.

no soundness is not based upon empricism,

StardustyPsyche said...

Joe Hinman said...

The flaw in 3. is the notion that imagining things in a certain way means they must be realizable in actual outside existence.

Premise 3. is simply unsound suffering from being a non-sequitur.

" not what it says"
Sorry, yep, it does.

Merely because a thought can be conceived without contradiction it does not follow that it must be not impossible.

" yes it does, Impossible in logo does not mean it doesn't exist or that it seems it wont work given the laws of physics, the world of logic is not ruled by empirical rules."
Ok, in the world of logical thought it can exist as a thought. A thought can exist as a thought. Congrats on the tautology.

"It means one thing only, impossible in logic means that it is a contradiction in logic.If there is no contradiction it's not impossible."
...in thought, agreed.


I can think of things that seem to be free of contradiction yet the reality of the universe may be that such a thought cannot be realized and is thus impossible.

" that's wrong for the reason i just said. it;s not abouit what works in the real world"
Agreed, the ontological argument has nothing to do with the real world of outside existence, it is purely an internal thought exercise.

" of physics it's logoc. only if it contradicts logically is it impossible."

" let's re,e,bert this comare up in relation to validity and that a question of logic,"
I realize you have publicly declared your are dyslexic, fine, but surely your spell checker alerts you to the errors that make your above sentence prohibitively difficult to decipher.



Line 3. is unsound, and thus the ontological argument is unsound.

" no soundness is not based upon empricism,"
For a proposition to be existentially sound it must comport with existential reality, not merely logical consistency.


January 15, 2017 10:33 PM

Joseph Hinman (Metacrock) said...

my challenge to Dusty on modal argument on CADRE blog

Fun iwth nodal argument

Joseph Hinman (Metacrock) said...

Merely because a thought can be conceived without contradiction it does not follow that it must be not impossible.

see link above I answer it there

StardustyPsyche said...

Joe Hinman said...

Merely because a thought can be conceived without contradiction it does not follow that it must be not impossible.

" see link above I answer it there"
What a ridiculous smokescreen of blather.

Just answer the points succinctly.

Line 3. is unsound because merely thinking of a thing in a way that is logically self consistent in thought does not mean it must be possible to realize outside of thought.

A logically self consistent thought can be impossible in reality.

The ontological argument is unsound as a real existential necessity.


January 16, 2017 12:13 AM

Kevin said...

SD: "That's what rational engagement is for, else it would merely be argument from authority.

In science there are no authorities."

We might be having two separate conversations, since I'm talking about any two random people on any random debate subject, not just science. Here's an attempt at rewording my question.

If an argument is presented to a group, and half agree it is a good argument and half reject it as a bad argument, then half the people are right and half are wrong. The difference between them, assuming lucky guesses are not a factor, is half the people recognized the argument's merit or lack thereof, and the others failed to recognize it.

So, how does a person determine which group he resides in? What is the functional difference between someone who is correct about an argument's merits, and someone who is wrong but unable to see that he is wrong?

Kevin said...

Disclosure: this isn't an attempt at a "gotcha" trap. I'm fascinated by how the mind works in the formation of beliefs and the acceptance of facts and propositions when filtered through pre-existing beliefs and biases. You seem to be approaching this forum as an observer of an argument's merit rather than a participant with an opinion of that merit, which is rather unique.

Joseph Hinman (Metacrock) said...

there is authority in science that's why they have degrees.it's propaganda to think science is all open thinking and searching and not authority, science has it's ideology like any the truth regime.

Joseph Hinman (Metacrock) said...

Dustry is afraid to meet me om my turf.


but he'[s going to have to. the modal argumnet is one of the big gun arguments it deserves a thread of it's own. use your your initiative Dusty. Read the answeron the link and answer it here.

Joseph Hinman (Metacrock) said...

My challenge to Dusty on Cadre

Joseph Hinman (Metacrock) said...

anyone who wants to know the answer read it on the CADRE blog please leave a note

Ilíon said...

bmiller's post of 2017/01/15 9:15 PM is a good illustration of how 'Science!' fetishizing 'atheists' (*) think (if I may be forgiven for using that word in this context).

Allow me to explain, in different words, what is going on in that exchange --

bmiller reminded Psychotic Dust of a necessary entailment of materialism (**), to wit: that all things -- including his and everyone else's actions and thoughts -- are mechanistically caused by prior physical states. That is, a person's belief (which is itself mechanistically caused by prior physical states) that he does action X for reason Y or that he has observed state Z are all illusions which are mechanistically caused by prior physical states.

And Psychotic Dust agreed.

bmiller *then* reminded Psychotic Dust that scientists -- and *all* the things they do and think -- are also included the all things that are mechanistically caused by prior physical states. And thus, a scientist's belief (which, once again, is itself mechanistically caused by prior physical states) that he has done experiment X by means of protocol Y and observed result Z are all illusions which are mechanistically caused by prior physical states.

And -- presented with the inescapable logical entailments of his God-denial -- Psychotic Dust did what *all* God-deniers *always* do in that situation: in a brazen display of intellectual dishonesty, he ascribed to himself (and to 'atheists' in general) the God-like ability to transcend the mechanistic causation of prior physical states.



(*) all of them, not just Psychotic Dust.

(**) materialism *just is* the concatenation of
-- affirmation of the reality of the physical/material;
-- denial of the reality of the Creator, that is, denial that the ontological basis of being, and of all-that-is, is an actually existing Mind;
or, to put it another way, materialism *just is* Western-style atheism. In contrast, Eastern-style atheism also denies reality the physical/material.

Joseph Hinman (Metacrock) said...

Line 3. is unsound because merely thinking of a thing in a way that is logically self consistent in thought does not mean it must be possible to realize outside of thought.


answered on the link go read it on the CADRE blog

A logically self consistent thought can be impossible in reality.

no it can't be. Impossible means no possibility it could ever be true in any possible world. If you have something doesn't exit in this world,say Bigfoot, let' say it could exist in some possible world. There no logical reasom why it can't exist, it just does't happen to.

If we deal with something that contradicts physical law there could still be a possible world in which the physical law is different.But in the case of an idea that is logically contradictory it could never the true. Take a square circle. A circle could never be square it's a logical contradiction.

In issues that are not logical contradictions they may be non existence in our world but they are not impossible. If the idea is not contradictory internally it is not possible then it is possible.

there are some more key concepts you have to read what wrote on thye loihnk to get them,


The ontological argument is unsound as a real existential necessity.

yes? what's an existential necessity? let's see you prove that statement can you even give me a reason? the modal argument is not really the ontological argumnet it;s very different and the difference means the old objections Kant made are gone,

bmiller said...

@Ilíon,

Exactly. Thanks for summarizing.

Let me highlight one other aspect of the exchange.

Science is based on the observation of cause and effect. If one comes to the conclusion that there are no causes or effects then one has disproven the validity of science, at least in one’s own mind. It is then a special type of crazy to use the thing you’ve concluded is invalid as the basis of argument against your opponents.

Unknown said...

Legion: "If an argument is presented to a group, and half agree it is a good argument and half reject it as a bad argument, then half the people are right and half are wrong. The difference between them, assuming lucky guesses are not a factor, is half the people recognized the argument's merit or lack thereof, and the others failed to recognize it. / So, how does a person determine which group he resides in? What is the functional difference between someone who is correct about an argument's merits, and someone who is wrong but unable to see that he is wrong?"

Consistency.

Which group is being consistent?

If one group says that argument supports a given belief, but they deny the force of that same argument for a belief they don't support, then we can identify which group is wrong. The group that is not consistent is wrong.

We call these inconsistencies "fallacies."

Kevin said...

"The group that is not consistent is wrong."

I would bet my income that there is no one who is consistent.

Unknown said...

Legion: "I would bet my income that there is no one who is consistent."

It's not about human fallibility; we are all fallible.

It's about being consistent with regard to the force of the same argument. Doing this doesn't grant one magical powers. But it does do what you ask -- identify which group is wrong.

Unknown said...

Legion: "Of course there are [sound arguments for a theistic god as a necessary being that have ever been published into general circulation]. Your failure to grasp them or recognize them is irrelevant to the fact that such arguments exist. They were presented to you in the other thread, over and over, and you remained utterly incapable of, or unwilling to, understand them."

Okay. Please copy and paste, from the other thread to, the "sound arguments for a theistic god as a necessary being that have ever been published into general circulation."

Kevin said...


"It's about being consistent with regard to the force of the same argument. Doing this doesn't grant one magical powers. But it does do what you ask -- identify which group is wrong."

I must not understand you, then. But that's my fault for not identifying a specific subject in the hypothetical situation, I suppose.

"Okay. Please copy and paste, from the other thread to, the "sound arguments for a theistic god as a necessary being that have ever been published into general circulation."

Why? They were already presented there. The objections were noted, and I rejected them as inadequate or irrelevant.

B. Prokop said...

Cal and Stardusty's error lies in demanding a "proof" that satisfies 100% of everyone, but most especially them personally. You're never going to get that for anything, be it "Is Jesus the Son of God?" or "Is the big bang theory correct?" or even "Did Lee Harvey Oswald act alone?" So stop asking for one. What you ought to be asking is why someone believes how they do, and then respond with what you believe. But at the end of the day, no one argument is ever going to please everyone.

I myself am totally convinced by the fact there is something rather than nothing, that time had a beginning, and the Gospel narratives are reliable accounts of actual events. That's more than enough for me. There exist many, many buttressing arguments for my faith, but that's basically all they are - suspenders and a belt as it were, but not essential.

And it matters not a whit to me that someone else remains unconvinced by what I see as reasonable and sufficient evidence. No more than I think OJ wasn't guilty as sin simply because 12 jurors decided otherwise.

B. Prokop said...

On this Martin Luther King, Jr. Day, I thought it might be appropriate to quote from his Letter from a Birmingham Jail, especially since some commenters here are so disparaging of St. Thomas Aquinas:

One may well ask, "How can you advocate breaking some laws and obeying others?" The answer is found in the fact that there are two types of laws: there are just laws, and there are unjust laws. I would agree with St. Augustine that "An unjust law is no law at all."

Now, what is the difference between the two? How does one determine when a law is just or unjust? A just law is a man-made code that squares with the moral law, or the law of God. An unjust law is a code that is out of harmony with the moral law. To put it in the terms of St. Thomas Aquinas, an unjust law is a human law that is not rooted in eternal and natural law. Any law that uplifts human personality is just. Any law that degrades human personality is unjust. All segregation statutes are unjust because segregation distorts the soul and damages the personality.


So much for Aquinas being "quaint" and "Medieval" and "irrelevant".

Saint Thomas Aquinas, ora pro nobis!

Kevin said...

"And it matters not a whit to me that someone else remains unconvinced by what I see as reasonable and sufficient evidence. No more than I think OJ wasn't guilty as sin simply because 12 jurors decided otherwise."

Well said.

Unknown said...

Legion: "What is the functional difference between someone who is correct about an argument's merits, and someone who is wrong but unable to see that he is wrong?"
Me: "If one group says that argument supports a given belief, but they deny the force of that same argument for a belief they don't support, then we can identify which group is wrong. The group that is not consistent is wrong.."
Legion: "I would bet my income that there is no one who is consistent."
Me: ""It's about being consistent with regard to the force of the same argument. Doing this doesn't grant one magical powers. But it does do what you ask -- identify which group is wrong."
Legion: "I must not understand you, then. But that's my fault for not identifying a specific subject in the hypothetical situation, I suppose."

I don't know how I could be any clearer in respect to answering your question.


Unknown said...

Legion: "Of course there are [sound arguments for a theistic god as a necessary being that have ever been published into general circulation]. Your failure to grasp them or recognize them is irrelevant to the fact that such arguments exist. They were presented to you in the other thread, over and over, and you remained utterly incapable of, or unwilling to, understand them."
Me: ""Okay. Please copy and paste, from the other thread to, the "sound arguments for a theistic god as a necessary being that have ever been published into general circulation."
Legion: "Why? They were already presented there. The objections were noted, and I rejected them as inadequate or irrelevant."

This seems like a dodge.

Why won't you just clarify, and copy and paste from the other thread the best of the arguments you were thinking of when you wrote the above when you declraed that Stardusty failed to grasp or recognize them.

Unknown said...

"And it matters not a whit to me that someone else remains unconvinced by what I see as reasonable and sufficient evidence. No more than I think OJ wasn't guilty as sin simply because 12 jurors decided otherwise."

The problem with this is that's exactly what the members of the OJ jury told themselves.

Kevin said...

"This seems like a dodge."

Were over 400 posts not enough? I post an argument from over there, you reject it as baseless, I dismiss your rejection as baseless, insults start flying, SD shows up and says the argument fails immediately, you quote him and say "I know right?", I say you both fail to understand the argument, and 400 posts later, not one person has changed his mind. Shall I cite the First Way?

Kevin said...

"The problem with this is that's exactly what the members of the OJ jury told themselves."

Are you saying people should not trust their own reasoning?

David Brightly said...

Today @ 4:47 AM:
bmiller reminded Psychotic Dust of a necessary entailment of materialism (**), to wit: that all things -- including his and everyone else's actions and thoughts -- are mechanistically caused by prior physical states.

And a jolly good thing that would be too. For if not we might well be in danger of thinking and doing all manner of nonsense.

Unknown said...

Me: "This seems like a dodge."
Legion: "Were over 400 posts not enough?"

I didn't ask you to count the posts; I asked you to paste the best of the arguments you somehow characterized as being one of the supposedly many sound arguments for a theistic god.

Instead of doing as I keep on asking, you seem to be avoiding the request, and pretend now that there are 400 posts that do as you claim.

As if.

Do you know apologists most likely avoid responding to a simple request like mine?

They understand that it is better to pretend that there are good arguments somewhere else, rather than to lift the rock and show the argument itself.

Because that's when it becomes painfully obvious that the argument can't survive what all good arguments must do -- they have to be valid, sound (avoid fallacies), and be supported by evidence.

Is that what you think the First Way does (still)?







Unknown said...

Me: "The problem with this is that's exactly what the members of the OJ jury told themselves."
Legion: "Are you saying people should not trust their own reasoning?"

Nope.

I'm saying that when people don't take precautions to test their biases, when they come to trust their intuitions over the harder work of analysis and diligence, they make the kind of mistakes we see every day.

See this blog as a prime example.

Kevin said...

"I didn't ask you to count the posts; I asked you to paste the best of the arguments you somehow characterized as being one of the supposedly many sound arguments for a theistic god."

You are deliberately being obtuse in pretending to not understand why I am pointing out the post count. We discussed the merits of arguments for God's existence (primarily the First Way but also others) for over 400 posts, and you did nothing but reject them. So tell me, how in the world is it a dodge or avoidance to not repeat what we just went through? I can answer that for you - it isn't. Atheists sure love to pretend they win when theists weary of the same tired objections, though.

"Instead of doing as I keep on asking, you seem to be avoiding the request, and pretend now that there are 400 posts that do as you claim...As if...Do you know apologists most likely avoid responding to a simple request like mine? They understand that it is better to pretend that there are good arguments somewhere else, rather than to lift the rock and show the argument itself. Because that's when it becomes painfully obvious that the argument can't survive what all good arguments must do -- they have to be valid, sound (avoid fallacies), and be supported by evidence."

None of this bears the slightest resemblance to reality, especially when you are expecting me to repeat the conversations we JUST HAD. Are you pretending to not know how absolutely stupid that would be?

"Is that what you think the First Way does (still)?"

If the weaknesses of the argument were summarized by you and Stardusty, then yes, I still do. Your objections were insufficient to refute it. Unless you have better objections that weren't already covered in that thread?

"I'm saying that when people don't take precautions to test their biases, when they come to trust their intuitions over the harder work of analysis and diligence, they make the kind of mistakes we see every day."

That is advice that you should perhaps turn inward.

Unknown said...

Legion: "Atheists sure love to pretend they win when theists weary of the same tired objections, though."

Or, they get tired of apologists pretending that they've offered good arguments elsewhere, but NEVER providing them in the here here.

Legion: "None of this bears the slightest resemblance to reality, especially when you are expecting me to repeat the conversations we JUST HAD. Are you pretending to not know how absolutely stupid that would be?"

When you make a false assertion (that you previously made sound arguments for a theistic god), it would be wrong of me to not point out that you are actually pretending.

Legion: "If the weaknesses of the argument were summarized by you and Stardusty, then yes, I still do."

The argument ends with the non-sequitur: ''... and this everyone understands to be god." which is also demonstrably false -- I am part of everybody ,and I DEFINITELY don't agree that any god, let alone the Christian one, is supposed to be doing tacked into that "argument."

So, to be clear -- you think that a false, non-sequitur is a sound conclusion to an argument. And that you think that that your refusal to recognize this puts us at an impasse.

But it doesn't create an impasse. It identifies you as belong to a group that refuses to be consistent regarding the force of argument.

That is not good company.

grodrigues said...

@Legion of Logic:

"So tell me, how in the world is it a dodge or avoidance to not repeat what we just went through? I can answer that for you - it isn't. Atheists sure love to pretend they win when theists weary of the same tired objections, though."

Especially when it is *demonstrably true* that they do not have the foggiest idea of how the argument is supposed to work. So their ignorant opinion about the argument is absolutely irrelevant -- if the argument indeed fails, it is a mere accident that they have hit upon the truth.

bmiller said...

@Legion of Logic,

"We discussed the merits of arguments for God's existence (primarily the First Way but also others) for over 400 posts, and you did nothing but reject them."

I disagree. We spent most of the 400+ posts explaining to SP that he was attacking the wrong argument. Want to bet we experience Ground Hog Day again if he re-engages?
What’s interesting is that they seem passionate about the subject, but seem unable to go beyond the first returned Google search for “how to refute the First Way”. Sad.

B. Prokop said...

This is rich. Cal and Stardusty's entire "refutation" of the First Way basically boils down to "It doesn't convince me." Sorry guys, but your non-acceptance of the argument in no way means the argument is not sound. It just means you don't like it. (Perhaps because acceptance would force you to "test your biases"? But like Gollum, you cannot bring yourself to abandon your precious atheism?)

StardustyPsyche said...

Legion of Logic said...

SD: "That's what rational engagement is for, else it would merely be argument from authority.

In science there are no authorities."

We might be having two separate conversations, since I'm talking about any two random people on any random debate subject, not just science. Here's an attempt at rewording my question.

If an argument is presented to a group, and half agree it is a good argument and half reject it as a bad argument, then half the people are right and half are wrong. The difference between them, assuming lucky guesses are not a factor, is half the people recognized the argument's merit or lack thereof, and the others failed to recognize it.

" So, how does a person determine which group he resides in? What is the functional difference between someone who is correct about an argument's merits, and someone who is wrong but unable to see that he is wrong?"
A typical functional difference is that people who work hard to identify and eliminate unsound argumentation in others and in themselves tend to not be on the bad argument side.

People who go around saying "I kicked your ass" or "you are an expletive" while repeatedly engaging in transparently fallacious argumentation typically are on the bad argument side.

Those are generalizations, of course, but I have found them to be highly correlated as described.


January 16, 2017 2:21 AM

StardustyPsyche said...

Ilíon said...
" And -- presented with the inescapable logical entailments of his God-denial -- Psychotic Dust did what *all* God-deniers *always* do in that situation: in a brazen display of intellectual dishonesty, he ascribed to himself (and to 'atheists' in general) the God-like ability to transcend the mechanistic causation of prior physical states."
Hmmm... I don't recall doing that exactly. This sounds like a strawman characterization of something I said.


January 16, 2017 4:47 AM

bmiller said...

@Legion of Logic,

For your reference from that thread:

“In context, Aquinas is instructing graduate Theology students and not just people off the street. So they would know the definition of God.”
He starts the demonstration with:
"I answer that, The existence of God can be proved in five ways."
and ends the demonstration with:
"and this everyone understands to be God."

If someone wants to contest that the latter phrase is part of the demonstration, then they must show that Aquinas intended to include the latter phrase as part of his demonstration. Let them try.

StardustyPsyche said...

Joe Hinman said...

" no it can't be. Impossible means no possibility it could ever be true in any possible world. If you have something doesn't exit in this world,say Bigfoot, let' say it could exist in some possible world. There no logical reasom why it can't exist, "
But it is not necessarily the case that it can possibly physically exist in any possible actual world.

You are confused between the limits of imagination and the limits of reality. Just because you can imagine a thing does not mean necessarily there is an actually possible real world wherein your thought is realizable.



" If we deal with something that contradicts physical law there could still be a possible world in which the physical law is different."
You don't know that necessarily merely based on your ability to imagine it.



" In issues that are not logical contradictions they may be non existence in our world but they are not impossible. "
You don't know that necessarily. They could be physically impossible. Merely thinking of the logical possibility does not demand the physical possibility.



"If the idea is not contradictory internally it is not possible then it is possible."
In your thoughts, but it does not follow that it must also be existentially possible.

" there are some more key concepts you have to read what wrote on thye loihnk to get them,"
Ha Ha Ha.

" The ontological argument is unsound as a real existential necessity.

yes? what's an existential necessity? let's see you prove that statement can you even give me a reason? "
I already have many times.

3. Whatever can be conceived without contradiction is not impossible.

This is an unjustified strong claim that is a non-sequitur. Merely conceiving of a thing that is free of logical contradiction does not guarantee its existential possibility. Reality could be such that the thought is impossible to realize in all actually possible worlds.


January 16, 2017 5:02 AM

SteveK said...

Possible worlds
https://infogalactic.com/info/Possible_world#Possibility.2C_necessity.2C_and_contingency

Joseph Hinman (Metacrock) said...

metacrock's blog

B. Prokop said...

Okay. Please copy and paste, from the other thread to, the "sound arguments for a theistic [G]od as a necessary being that have ever been published into general circulation."

All right. I will try one more time.

The best (and unassailable) argument for the truth of Christianity is the reliability of the Gospels as an accurate narrative of real world events. Christianity (and I care not a whit for some phantom labeled "theism") stands or falls on the literal truth of the Gospel. I for one am willing to take my stand (and indeed, stake my life) on this line.

So take your best shot. Show to me why I should not believe the Gospels to be true. No links, no "copy/paste", no "read this book" - just tell me "here and now" why you believe them to be false.

Joseph Hinman (Metacrock) said...
This comment has been removed by the author.
Joseph Hinman (Metacrock) said...

the URL Steven K posted on possible worlds

here

Joseph Hinman (Metacrock) said...


Ironically Stardust, like so many atheists on the net, really fit the stereo type he imagines of all religious people, Scared to death to move beyond the boundaries o what he knows, He is totally insulated against being convinced because he is closed to any source of information or thinking but the atheist echo chamber.

He does not now the basics of science or logic. he thinks "impossible" so defined by science he doesn't get that science is not dedication. He doesn't understand why science is called "empirical." or the difference between empirical knowledge and deductive logic he's not about to find out. if it doesn't back the God hate it's mot worth knowing.

of things that doesn't exist there are two kinds, There are things that don't exist because they can't since they are impossible and there are things that don't happen to exist because the world doesn't stack up in such a way as to make it so.Of that possibility there are two reasons, either because it just didn't happen or because the laws of physics are such tat it can't happen given our current world, those are still possibilities,a logical impossibility is one that is a logical contradiction it could never be regardless of the nature of the world. that is what im eant by "in o possible world"

To Dusty this is just imagining because it's not observing something,he knows that scientists observe things.therefore in his view the only valid from of knowledge is observation. Of course how could we observe something that can't possibly exist? it's not imagining because it's a logical reality that impossibilities can't exist.Another way to say that is if the world was different it still would not or, "in no possible world."

when I say God is conceivable without contradistinction I mean without logical contradiction.We can have a concept of God that snot a contradiction in terms, Thus its not impossible because.

we are coming upon 200 posts we cam't keep this up,we have different conversations, you need to come to the cadre log and argue it over there. I explaimn all of this over there.

Joseph Hinman (Metacrock) said...

Stardusty Psyche said...
Joe Hinman said...

" no it can't be. Impossible means no possibility it could ever be true in any possible world. If you have something doesn't exit in this world,say Bigfoot, let' say it could exist in some possible world. There no logical reasom why it can't exist, "
But it is not necessarily the case that it can possibly physically exist in any possible actual world.

You are confused between the limits of imagination and the limits of reality. Just because you can imagine a thing does not mean necessarily there is an actually possible real world wherein your thought is realizable.

you don't know shit about logic. this is not imagination just because it's not based upon observing a concrete thing. we can't observe logical contradictions in real life. They don't exist, they can't exist. we can still know about them through reason, that is not imagining it's logical you can work it out on a vain diagram,


" If we deal with something that contradicts physical law there could still be a possible world in which the physical law is different."

You don't know that necessarily merely based on your ability to imagine it.


yes we do .all scientists agree,. no scientist anywhere says logical contraindications might be true in the real world, Scientists disagree about the nature of laws of physics too. In fact the concept of physical law in modern science comes from God..


Joseph Hinman (Metacrock) said...

Dusty:

" In issues that are not logical contradictions they may be non existence in our world but they are not impossible. "

You don't know that necessarily. They could be physically impossible. Merely thinking of the logical possibility does not demand the physical possibility.

read the words Einstein I said they may be impossible in this world but in another world they night not be The laws of physics could be different in different worlds all scientists agree that is possible.But a logical contradiction is always impossible in any world. God gave you a brain turn it on! you cannot find a scientist who disagrees with I just said about logical contradictions.


explaining these elementary things to you is wasting my time I have real writing to vet on with.


grodrigues said...

@SteveK:

Books are where we can learn the real deal. Pruss's "Actuality, Possibility, and Worlds" is very good (like everything else from him) on the philosophical side. For the nitty-gritty mathematical stuff, Hughes and Cresswell "A New Introduction to Modal Logic" is a good option.

Kevin said...

"If someone wants to contest that the latter phrase is part of the demonstration, then they must show that Aquinas intended to include the latter phrase as part of his demonstration. Let them try."

Yep.

"All right. I will try one more time."

Definition of insanity! :) Good luck. Though I suspect at worst you'll have Richard Carrier thrown at you, shouldn't be too difficult.

B. Prokop said...

Legion,

Just a case of "sauce for the goose is sauce for the gander."

"Carrier ... shouldn't be too difficult"

A fine example of understatement there!

To be up front, there are indeed some difficult passages in the Gospels, and some legitimate questions about reconciling differences between the 4 accounts, no doubt. But I've yet to see a "problem" that doesn't evaporate upon sufficient study, or by reexamining one's going-in premises.

For instance, a common objection to the reliability of the Gospels is that the precise wording of some of the things Jesus says differs from one version to another. The skeptics will say, "Aha! A contradiction!" But we have to keep in mind that, with rare exception, we do not know the original words of Christ. What we have for the most part are Greek translations of what was originally spoken in Aramaic. Anyone who's worked in the field knows well that you can have multiple renderings, all equally accurate, after translation. I ought to know. For years, I was a paid translator for The American Institute of Physics, translating the works of Russian physicists into English. (very boring work, by the way)

StardustyPsyche said...

B. Prokop said...

" Okay. Please copy and paste, from the other thread to, the "sound arguments for a theistic [G]od as a necessary being that have ever been published into general circulation."


" The best (and unassailable) argument for the truth of Christianity is the reliability of the Gospels as an accurate narrative of real world events.

So take your best shot. Show to me why I should not believe the Gospels to be true. No links, no "copy/paste", no "read this book" - just tell me "here and now" why you believe them to be false."

Fair enough!!! Whatever our differences may be we clearly share a disdain for certain common blogging practices!!!

My claim that "there are no sound arguments for a theistic [G]od as a necessary being that have ever been published into general circulation." was carefully worded and the gosples certainly are not a counter example to my particular claim.

But from your other posts I doubt you are asserting the gospels as a counter example to my particular claim, rather, you seem to be arguing from a personal level of acceptance, that is just makes sense to you on a personal basis and that is good enough for you.

So for you, fine, believe whatever you want to believe, that is up to you. But for me the gospels are preposterous, utterly unbelievable, totally underwhelming. From a rational argumentation standpoint they certainly are not a sound argument for god as a necessary being, which is my claim stated in a number of posts above.

The gospels are unsound for so many reasons I will not attempt to be comprehensive but argument from authority, non-sequitur, and false dichotomy abound.

The gospels are just mundane god fables like thousands of others. Most such fables last for a time and then fade into obscurity. A few have caught hold and have gotten entrenched in the culture of the credulous masses. Hindu, Islamic, and spiritual versions of Buddhism are others.

The stories themselves are preposterous and of no more believability than tabloid fiction. The gospels are not even consistent between themselves, so much so that Christians go through apologetic mental gymnastics to try to reconcile the various tellings of the same story.

The struggle to comprehend these supposed works of the almighty is never ending. On the one hand Jesus affirmed the law, yet he contradicted parts of it explicitly, and with his new message of love and forgiveness he contradicted vast numbers of the over 600 commandments implicitly, leading to endless debates that are never settled. How believable is it that the omnipotent omniscient creator of the universe inspired such a muddled work? I could fix the lack of clarity of the gospels in a few days work and so could you.

Jesus clearly got rid of the old writ of divorce with his new message of love, yet the gospels again lead only to endless fighting among Christians. Is there no exception or is infidelity an exception? The debate rages on because this work of the perfect being is so badly written.

The miracle stories are childish. Further, even if a guy could walk on water that does nothing logically to demonstrate he is the creator of the universe, all good, all knowing, or any of the other asserted properties.

There have been many god men, often leading their devote followers to their deaths. Claims to be god abound, as do legions of followers who believe it devoutly. People are credulous by nature. You, for example.


January 16, 2017 10:31 PM

B. Prokop said...

Before we proceed, let me make sure I understand your objections in the same way you do. You say:

1. The Gospels are "preposterous" - they deal with impossible events.

2. The Gospels are "just like" many similar "fables".

3. The four Gospels are not consistent with each other.

4. The Gospels are irreconcilable with Jewish Law.

5. The miracles of Christ are not proof of His divinity.

6. The Gospels are "badly written".

7. The Gospels are "childish" and only believable to the "credulous".

Have I represented your views accurately?

Joseph Hinman (Metacrock) said...

Prokop Jews of Jesus' time spoke Greek as often or more often than Aramaic.

Joseph Hinman (Metacrock) said...

" Okay. Please copy and paste, from the other thread to, the "sound arguments for a theistic [G]od as a necessary being that have ever been published into general circulation."

sound means valid plus all premises true, you have not shown a false premise to any argument especially not mine not mine,

B. Prokop said...
This comment has been removed by the author.
B. Prokop said...

"Jews of Jesus' time spoke Greek as often or more often than Aramaic."

Correct for the Jews of the Diaspora. But for Judeans and Galileans, Aramaic was the Mother Tongue.
(I agree with you that Jesus was most likely fluent in Greek, based on John's account of the trial before Pilate. But in "regular life" the overwhelming probability is that He spoke Aramaic.)

B. Prokop said...

Assuming I interpreted your objection correctly, I decided to go ahead and begin answering them.

Seven objections are too many to deal with adequately all at once, so allow me to restrict myself to your first three for now - two in this posting, and the third in the one immediately following. (We’ll hit the others later.)

1. The Gospels are "preposterous" - they deal with impossible events.

What you are most likely really reacting to here is the presence of the miraculous. (Please correct me if I’m wrong.) You have probably defined a miracle as something along the lines of “an action that breaks or contradicts the laws of nature”, and go from there to categorize them as “impossible” events. So your objection is not to the Gospels per se, but to their inclusion of miracles. Fair enough, but really an entirely different discussion, namely “Are there such things as miracles?” Until you can say for certainty that miracles are impossible, your objection does not hold up as anything pertaining to the historical reliability of the Gospels.

Were I to tell you that the Battle of Lepanto was won by the Venetians (a.k.a., the Holy League) due to the direct intervention of the Blessed Virgin Mary (which, by the way, I do believe), you could not object to that solely on the grounds that my account includes a miracle, unless you first proved that miracles do not and cannot occur.

2. The Gospels are "just like" many similar "fables".

Actually, that is the last thing they are like. It takes but a glance at the Gospels to see the enormous differences, and the great number of them. I happen to be a huge fan of Greek Mythology. I list Homer amongst my all time favorite writers. I read the tragedians for fun. I know what mythology (what you call “fables”) looks like, and the Gospels simply do not fit the description.

The Gospels are firmly grounded in history, pinned to clearly (indeed precisely) identified times and places. They are chock-a-block with names, locations, events, and activities which are Real World. You can even today see with your own eyes the Pool at Siloam, the Stone upon which Pilate’s seat was located, Jacob’s Well in Samaria, beside which Jesus rested in John Chapter 4, the very cemetery on the shore of the Sea of Galilee, in which Jesus expelled the demons from the man “living in the tombs”, the Garden of Gethsemane where Jesus was arrested. These are actual places. There is not the slightest hint of imaginary landscapes, cities in the clouds, or far off, unlocated (and unlocatable) islands, such as we encounter in The Odyssey. Such situational vagueness is baked into the DNA of mythology. Its stock in trade is “long, long ago, and far away”.

Of even greater contrast, we see no human beings turning into trees, birds, or constellations. There are no personified natural features such as rivers (as in The Iliad) or even the Earth and Sky (as in Hesiod’s Theogony). In fact, from a purely literary standpoint, absent the miracles of Christ, the world of the Gospels is about as realistic as anything written by Jane Austen or Ernest Hemingway (if not more so).

B. Prokop said...

(continued)

3. The four Gospels are not consistent with each other.

This particular objection of yours is probably the closest thing you have to a good point. Unfortunately, you immediately messed up by poisoning the well to any explanatory counter-argument. You’ve preemptively labeled anything that contradicts your supposition as “apologetic mental gymnastics”. How convenient. No need to engage the substance – just put in in a box labeled “do not seriously consider” and done!

I actually consider the minor inconsistencies between the 4 Gospels to be strong evidence in favor of their historical accuracies. Why? Well, if the Gospels were word for word identical, then why bother having 4 of them? Also, these are eyewitness accounts. Every competent police detective knows that if 2 witnesses are too much in agreement with each other, then their stories are likely cooked – the product of collusion. But if one guy says the car was a dark blue Hyundai and another thinks it might have been a green Toyota, they’re both probably telling the truth.

And the interesting thing about every one of the supposed inconsistencies is… (wait for it)… their utter insignificance. None of them involve any major facet of the story being told – Matthew recounts the temptations in the wilderness in one order, Luke in another – but both accounts agree on everything of any importance. In Matthew, a Roman centurion asks Jesus to come and heal his servant. In Luke, the centurion sends some Jewish representatives to make the request. But both Gospels agree on all the essential points of the event, differing only in inconsequential minutae. In Mark, Jesus responds to a demand for a sign with, “Why does this generation seek a sign? Truly I say to you, no sign shall be given to this generation.” In Matthew, the same response reads as follows: “An evil and adulterous generation seeks for a sign; but no sign shall be given to it except the sign of the prophet Jonah.” If that’s what worries you in the way of inconsistency… well, there’s an old saying about missing the forest for the trees that might have some applicability here.

StardustyPsyche said...

Joe Hinman said...

" when I say God is conceivable without contradistinction I mean without logical contradiction."
Exactly.

3. Whatever can be conceived without contradiction is not impossible.

The prefix "im" means not in this context, so the statement is precisely equal to

3. Whatever can be conceived without contradiction is not not possible.

Which is precisely equal to

3. Whatever can be conceived without contradiction is possible.

Are you referring to a logical "possible" or an existential "possible"?

If you are referring to a logical "possible" then the Ontological Argument is a mere logical tautology that says nothing about what exists outside of human thought.

If you are referring to an existential "possible" then line 3. is a non-sequitur rendering the Ontological Argument unsound as an argument for the real existence of god.

Either way, the Ontological Argument is of no existential value.


January 16, 2017 11:42 PM

StardustyPsyche said...

B. Prokop said...
" Were I to tell you that the Battle of Lepanto was won by the Venetians (a.k.a., the Holy League) due to the direct intervention of the Blessed Virgin Mary (which, by the way, I do believe), you could not object to that solely on the grounds that my account includes a miracle, unless you first proved that miracles do not and cannot occur."
Nope, you have that backwards. Your assertion, your burden.

" 2. The Gospels are "just like" many similar "fables".

Actually, that is the last thing they are like. It takes but a glance at the Gospels to see the enormous differences,"
Nope, one fairy tale is basically the same as another. Just a lot of story telling about magical beings.


" The Gospels are firmly grounded in history,"
So is Gone With The Wind, which at least is about fairly believable events, but is honest enough to make no pretense of actually having occurred, whereas the bible is a fable of bizarre magical events that absurdly tries to pass itself off as true historical facts.


January 17, 2017 8:45 PM

Joseph Hinman (Metacrock) said...


" when I say God is conceivable without contradistinction I mean without logical contradiction."
Exactly.

3. Whatever can be conceived without contradiction is not impossible.

The prefix "im" means not in this context, so the statement is precisely equal to

3. Whatever can be conceived without contradiction is not not possible.

Which is precisely equal to

3. Whatever can be conceived without contradiction is possible.

that's dyslexia for you. It should say with contradiction .auto correct changed it to contradistinction and so i didn't notice becasue I expected it to say contradiction and dyslexia worked against noticing



Are you referring to a logical "possible" or an existential "possible"?

MA turns o what Plantinga calls "broadly logical

If you are referring to a logical "possible" then the Ontological Argument is a mere logical tautology that says nothing about what exists outside of human thought.


wrong. don't work that way,broadly logical is not just truth by definitions like all husbands are married men. But it's not empirical either.Even If it was the case it would not be true. Logical contradiction pertains to conceiving of God not to the entire argument, Since God can't be impossible by natural law because he's not subject to natural law since he created it, he is only subject to logic.

If you are referring to an existential "possible" then line 3. is a non-sequitur rendering the Ontological Argument unsound as an argument for the real existence of god.

first of all I don't believe you know whist an existential possibility is you dotcom understand the distinction between empirical and deductive you sure don't know what existentialist means.

Moreover the whole argument turns on the issue that God can't be a possibility. he is either on or off, either necessary or impossible he can't be a merely possible option. so that's just a contradction in your thinking, a category mistake.



Joseph Hinman (Metacrock) said...

Dusty you need to get it through your head, this is not the OA it's the MA very different, not sueject to the criticisms of Kant or Russell.

StardustyPsyche said...

Joe Hinman said...
" Since God can't be impossible by natural law because he's not subject to natural law since he created it, he is only subject to logic."
Ad hoc definition. Defining god into existence is just begging the question.

If god has always existed she did not create herself. The nature of her existence must be inherent in the nature of existence itself. Can existence allow for a god? Is god existentially possible? Line 3. asserts that if we humans can think of god then she must be existentially possible as opposed to an existence that does not allow for a god to have always existed.

Existential possibility does not follow from logical possibility, and 3. is therefore a non-sequitur.


" Moreover the whole argument turns on the issue that God can't be a possibility. "
Wrong. Line 3. states that god must be possible. That is a strong claim that the nature of existence must be such to allow for a god that has always existed, which does not follow from mere human thought.


January 18, 2017 12:38 AM

«Oldest ‹Older   1 – 200 of 287   Newer› Newest»